Pretest Pediatrics

Réussis tes devoirs et examens dès maintenant avec Quizwiz!

2. An infant can regard his parent's face, follow to the midline, lift his head from the examining table, smile spontaneously, and respond to a bell. He does not yet regard his own hand, follow past the midline, nor lift his head to a 45° angle off the examining table. Which of the following is the most likely age of the infant?

a. 1 month

Questions 231 to 234 231. Place the patient on a ventilator with an FIO2 of 1.0.

a. 6.92 101 19 15

497. Injuries almost exclusively related to the shoulder, including rotator cuff tendinitis

a. A 15-year-old competitive swimmer

388. Dark field microscopy

a. A 15-year-old sexually active girl with a single non-tender lesion on her labia and regional nontender lymphadenopathy

81. Vitamin C

a. A 4-year-old with diarrhea, abdominal pain, and kidney stones which prove to be caused by calcium oxalate.

342. PHACE syndrome

a. A 6-month-old with blindness on the same side as a large facial lesion

431. A 13-year-old comes to your office expressing concern about his height. He had first seen you a year prior for his routine checkup and a preparticipation sports physical for soccer (see growth curve). Now in the eighth grade, all of his friends are taller than he is, and he is at a disadvantage on the soccer field playing against much larger boys. After obtaining height information from his parents shown here, you order a skeletal bone age radiograph. Which of the following results would allow you to assure him of an excellent prognosis for normal adult height?

a. A bone age of 9 years

30. You find a discrete, whitish polyp that extends through the tympanic membrane in a child with a history of recurrent otitis media. This most likely represents which of the following?

a. A cholesteatoma

265. A 7-day-old premature infant born at 26 weeks of gestation now has a grossly bloody stool, abdominal distention, and increasing oxygen requirements.

a. Abdominal series

222. A 13-year-old patient with sickle-cell anemia presents with respiratory distress; she has an infiltrate on chest radiograph. The laboratory workup of the patient reveals the following: hemoglobin 5 g/dL; hematocrit 16%; white blood cell count 30,000/μL; and arterial blood (room air) pH 7.1, PO2 35 mm Hg, and PaCO2 28 mm Hg. These values indicate which of the following?

a. Acidemia, metabolic acidosis, respiratory alkalosis, and hypoxia

99. Liver disease

a. All fat-soluble vitamins

483. You are the sideline physician for a local high school football team. During a district playoff game, the starting quarterback is sacked for a loss on third down. As the punter heads out onto the field, the quarterback is slow to come to the sidelines. He seems confused and dazed. Aside from his confusion, his examination is normal. After 10 minutes, he is lucid and wants to get back into the game. Based on published guidelines, which of the following is your correct course of action?

a. Allow the player back in the game

300. A 2-year-old patient arrives late to your office with his father and a sign-language translator. They are very apologetic, but the father communicates that he had car trouble at his dialysis center and thus was late picking up the child from day care. The father is concerned about his child's having intermittent red, bloody-looking urine. A gross inspection of the child's urine in your office looks normal, but the dipstick demonstrates 3+ blood. Which of the following is the most likely cause of this child's hematuria?

a. Alport syndrome

11. A previously healthy 5-year-old boy has a 1-day history of low-grade fever, colicky abdominal pain, and a rash. He is well-appearing and alert. His vital signs, other than a temperature of 38°C (100.5°F) are completely normal. A diffuse, erythematous, maculopapular, and petechial rash is present on his buttocks and lower extremities, as shown in the photograph. He has no localized abdominal tenderness or rebound; bowel sounds are active. Laboratory data demonstrate Urinalysis: 30 red blood cells (RBCs) per high-powered field, 2+ protein Stool: Guaiac positive Platelet count: 135,000/μL These findings are most consistent with which of the following?

a. Anaphylactoid purpura

182. During a regular checkup of an 8-year-old child, you note a loud first heart sound with a fixed and widely split second heart sound at the upper left sternal border that does not change with respirations. The patient is otherwise active and healthy. Which of the following heart lesions most likely explains these findings?

a. Atrial septal defect (ASD)

75. After helping his father in the yard, a 14-year-old boy complains of weakness and feels like his muscles are twitching. He begins to drool, and then collapses in a generalized tonic-clonic seizure. Upon the arrival of EMS, his heart rate is found to be 40 beats per min and his pupils are pinpoint

a. Atropine and pralidoxime (2-PAM)

382. Warm, red, tender axillary lymph nodes in a 5-year-old girl; she has a few red papules on her hand at the site of a feline scratch

a. Bartonella henselae

223. A 24-year-old woman arrives in the emergency center in active labor. She is at term, but received no prenatal care after 16 weeks of gestation when she lost her insurance coverage. The mother has an uncomplicated vaginal delivery. You are paged shortly after birth when the baby is noted to have respiratory distress. The infant has diminished breath sounds on the left, and the PMI is shifted toward the right. A chest radiograph is shown. The NG tube you placed earlier reveals the stomach to be below the diaphragm. Which of the following is the most likely diagnosis at this point?

a. Congenital cystic adenomatoid malformation

199. You are called to the nursery to evaluate a newborn. The mother has a history of systemic lupus erythematosus and gestational diabetes. The nurses are concerned because the baby has developed petechiae and bruising after his bath. Vital signs have been stable, with a heart rate in the 60 beats per minute range and a respiratory rate in the 40 breaths per minute range. You note a large liver, scattered petechiae, and an erythematous rash on the cheeks and on the bridge of the nose.

a. Congenital heart block

144. The mother of a 2-week-old infant reports that since birth, her infant sleeps most of the day; she has to awaken her every 4 hours to feed, and she will take only an ounce of formula at a time. She also is concerned that the infant has persistently hard, pellet-like stools. On your examination you find an infant with normal weight and length, but with an enlarged head. The heart rate is 75 beats per minute and the temperature is 35°C (95°F). The child is still jaundiced. You note large anterior and posterior fontanelles, a distended abdomen, and an umbilical hernia. This clinical presentation is likely a result of which of the following?

a. Congenital hypothyroidism

121. Two new mothers are discussing their infants outside the neonatal intensive care unit. Both were born at 36 weeks' gestation. One infant weighs 2600 g (5 lb, 12 oz) while the other infant weighs 1600 g (3 lb, 8 oz). The mother of the second infant should be told that her child is more likely to have which of the following conditions?

a. Congenital malformations

158. A newborn has hydrocephalus, chorioretinitis, intracranial calcifications, and anemia.

a. Congenital toxoplasmosis

237. An 8-year-old boy presents to your office for a second opinion. He has a 2-year history of intermittent vomiting, dysphagia, and epigastric pain. His father reports he occasionally gets food "stuck" in his throat. He has been on a proton pump inhibitor for 18 months without symptom relief. His past history is significant only for eczema and a peanut allergy. Endoscopy was performed 6 months ago; no erosive lesions were noted and a biopsy was not performed. You arrange for a repeat endoscopy with biopsy. Microscopy on the biopsy sample reveals many eosinophils. Treatment of this condition should include which of the following?

a. Corticosteroids

Questions 492 to 495 492. A 16-year-old female has a positive rapid plasma reagin (RPR) of 1:64 and an ulcer on her labia.

a. Dark-field microscopic examination

169. An infant has fusion of the eyebrows, heterochromic irises, a broad nasal root with lateral displacement of the medial canthi, and a white forelock.

a. Deafness

77. A 2-year-old boy found a bottle of his mother's prenatal vitamins and consumed the majority of them. He now has hematemesis and abdominal pain. He is febrile, and laboratory tests reveal a leukocytosis and hyperglycemia

a. Deferoxamine mesylate

323. A 6-year-old boy had been in his normal state of good health until a few hours prior to presentation to the ER room. His mother reports that he began to have difficulty walking, and she noticed that he was falling and unable to maintain his balance. Which of the following is the most likely cause for his condition?

a. Drug intoxication

34. A 20-month-old child is brought to the ED because of fever and irritability and refusal to move his right lower extremity. Physical examination reveals a swollen and tender right knee that resists passive motion. Which of the following is the most likely to yield the diagnosis in this patient?

a. Examination of joint fluid

16. A 10-year-old child arrives with the complaint of new-onset bed-wetting. He has had no fever, his urine culture is negative, and he has had no new stresses in his life. He is well above the 95th percentile for weight as is much of his family. Which of the following is most helpful in making a diagnosis?

a. Fasting plasma glucose of 135 mg/dL

400. A 2950-g (6.5-lb) black baby boy is born at home at term. On arrival at the hospital, he appears pale, but the physical examination is otherwise normal. Laboratory studies reveal the following: mother's blood type A, Rh-positive; baby's blood type O, Rh-positive; hematocrit 38%; and reticulocyte count 5%. Which of the following is the most likely cause of the anemia?

a. Fetomaternal transfusion

62. Megaloblastic anemia, glossitis, pharyngeal ulcers, impaired immunity

a. Folate deficiency

208. A previously healthy, active, 18-month-old African American child presents with unilateral nasal obstruction and foul-smelling discharge. The child's examination is otherwise unremarkable. Which of the following is the most likely diagnosis?

a. Foreign body

55. A teen, just back from a skiing trip, has blistering and peeling of several areas on her face; she reports the lesions started as firm, cold, white areas that felt stinging at the time and are now more sensitive than the surrounding skin.

a. Frostnip

94. A 14-year-old child has headache, hypertension, edema, and a change in urine output and color

a. Glomerulonephritis

440. An infant is brought to a hospital because her wet diapers turn black when they are exposed to air. Physical examination is normal. Urine is positive both for reducing substance and when tested with ferric chloride. This disorder is caused by a deficiency of which of the following?

a. Homogentisic acid oxidase

478. A 15-year-old girl is brought to the pediatric emergency room by the lunchroom teacher, who observed her sitting alone and crying. On questioning, the teacher learned that the girl had taken five unidentified tablets after having had an argument with her mother about a boyfriend of whom the mother disapproved. Toxicology studies are negative, and physical examination is normal. Which of the following is the most appropriate course of action?

a. Hospitalize the teenager on the adolescent ward.

445. A small-for-gestational-age infant is born at 30 weeks' gestation. At 1 hour of age, his serum glucose is noted to be 20 mg/dL (normally greater than 40 mg/dL). Which of the following is the most likely explanation for hypoglycemia in this infant?

a. Inadequate stores of nutrients

240. A 15-year-old girl is admitted to the hospital with a 6-kg weight loss, bloody diarrhea, and fever that have occurred intermittently over the previous 6 months. She reports cramping abdominal pain with bowel movements. She also reports secondary amenorrhea during this time. Stool cultures in her physician's office have shown only normal intestinal flora. A urine pregnancy test was negative, while an erythrocyte sedimentation rate (ESR) was elevated. Her examination is significant for the lack of oral mucosal ulcerations and a normal perianal examination. Anti-Saccharomyces cerevisiae antibodies (ASCA) are negative, while anti-neutrophil cytoplasm antibodies (p-ANCA) are positive. You confirm your presumptive diagnosis with a rectal biopsy. In counseling her about her disease, which of the following statements would be true?

a. Inheritance is autosomal dominant

174. Previous premature infant born at 27 weeks' gestation and now 6 months of age presents with macrocephaly and hydrocephalus on ultrasonogram

a. Intraventricular hemorrhage

211. A fully immunized 2-year-old presents to the emergency room with several days of low-grade fever, barking cough, and noisy breathing. Over the past few hours he has developed a fever of 40°C (104°F) and looks toxic. He has inspiratory and expiratory stridor. The family has not noticed drooling, and he seems to be drinking without pain. Direct laryngoscopy reveals a normal epiglottis. The management of this disease process includes which of the following?

a. Intubation and intravenous antibiotics

129. At 43 weeks' gestation, a long, thin infant is delivered. The infant is apneic, limp, pale, and covered with "pea soup" amniotic fluid. Which of the following is the best first step in the resuscitation of this infant at delivery?

a. Intubation and suction of the trachea; provision of oxygen

480. A 17-year-old sexually active girl comes to your office complaining of acne that is unresponsive to the usual treatment regimen. Physical examination reveals severe nodulocystic acne of her face, upper chest, and back. You consider prescribing isotretinoin (Accutane), but you are concerned about side effects. Reviewing the literature, you find which of the following to be true about isotretinoin?

a. Its efficacy can be profound and permanent.

4. A 4-year-old girl is noticed by her grandmother to have a limp and a somewhat swollen left knee. The parents report that the patient occasionally complains of pain in that knee. An ophthalmologic examination reveals findings as depicted in the photograph. Which of the following conditions is most likely to be associated with these findings?

a. Juvenile rheumatoid arthritis

397. After being delivered following a benign gestation, a newborn infant is noted to have a platelet count of 35,000/μL, decreased fibrinogen, and elevated fibrin spilt products. On examination you note a large cutaneous hemangioma on the abdomen that is purple and firm. Which of the following anomalies might also be expected in this infant?

a. Kaposiform hemangioendothelioma

68. A 6-year-old boy has developed a limp and has limited mobility of the hip, but denies pain and fever.

a. Legg-Calvé-Perthes disease

Questions 447 to 451 447. Three boys in a family with hypodense calcification of their bones and a history of fractures; their condition is unresponsive to dietary changes or vitamin supplementation

a. Low PO4, normal Ca

257. The mother of a 6-month-old infant is concerned that her baby may be teething. You explain to her that the first teeth to erupt in most children are which of the following?

a. Mandibular central incisors

317. A 9-year-old girl is brought by her sister to her pediatrician with the complaint of severe, intermittent headaches for the past several months, one of which resulted in her going to the ER. The physical examination today, including a careful neurologic examination, is normal. The headache is diffuse, throbbing, lasts several hours, and is not associated with vomiting or other symptoms. The child cannot feel the headaches coming on; they appear on all days of the week; and usually the headaches are gone when she awakens from a nap. The child reports that she is doing well in school, plays clarinet in the school band, and has "lots of friends." The sister is not sure, but she thinks their father, who lives in another state, may have headaches. The most likely explanation for this girl's headache is which of the following?

a. Migraine

92. A 4-month-old presents with a dry, scaly rash on his cheeks, arms, and upper chest. His 10-year-old sister had a similar rash when she was young, but the rash is now confined to her antecubital and popliteal fossa; her rash worsens in winter months

a. Mild cleansing cream, topical moisturizers, and topical steroids

Questions 471 to 474 471. A 3-year-old boy with myoclonus, ataxia, weakness, and seizures who has cytochrome oxidase-negative ragged red fibers noted on muscle biopsy

a. Mitochondrial inheritance

203. A previously well 1-year-old infant has had a runny nose and has been sneezing and coughing for 2 days. Two other members of the family had similar symptoms. Four hours ago, his cough became much worse. On physical examination, he is in moderate respiratory distress with nasal flaring, hyperexpansion of the chest, and easily audible wheezing without rales. His chest radiographs are shown. Which of the following is the appropriate next course of action?

a. Monitoring oxygenation and fluid status alone

390. A 14-year-old girl with low-grade fever, cough of 3 weeks' duration, and interstitial infiltrate.

a. Mycoplasmal pneumonia

184. An ill-appearing 2-week-old baby girl is brought to the emergency room. She is pale and dyspneic with a respiratory rate of 80 breaths per minute. Heart rate is 195 beats per minute, heart sounds are distant, a gallop is heard, and she has cardiomegaly on x-ray. An echocardiogram demonstrates poor ventricular function, dilated ventricles, and dilation of the left atrium. An ECG shows ventricular depolarization complexes that have low voltage. Which of the following is the most likely diagnosis based on this clinical picture?

a. Myocarditis

462. An 8-year-old boy with Addison disease (in crisis)

a. Na+ 118, K+ 7.5

Questions 458 to 463 458. A 4-month-old boy with salt-losing 21-hydroxylase deficiency (adrenogenital syndrome)

a. Na+ 118, K+ 7.5

305. Elevated levels of cholesterol and triglycerides found in a 6-year-old boy whose mother reports that he has been awakening with puffy eyes each morning. On your physical examination you determine that he has had unexpected weight gain and has scrotal edema

a. Nephrotic syndrome

Questions 49 to 53 49. A 3-year-old boy awakens every night around 2:00 AM screaming incoherently. His parents note that he is agitated, seems awake but unresponsive, and goes back to sleep within a few minutes. He has no memory of the episodes in the morning

a. Night terrors

425. A 6-year-old boy is brought to your practice by his paternal grandmother for his first visit. She has recently received custody of him after his mother entered the penal system in another state; she does not have much information about him. You note that the child is short for his age, has downslanting palpebral fissures, ptosis, low-set and malformed ears, a broad and webbed neck, shield chest, and cryptorchidism. You hear a systolic ejection murmur in the pulmonic region. His grandmother reports that he does well in regular classes, but has been diagnosed with learning disabilities and receives speech therapy for language delay. His constellation of symptoms is suggestive of which of the following?

a. Noonan syndrome

295. A 12-year-old boy comes to the emergency department at midnight with a complaint of severe scrotal pain since 7 PM. There is no history of trauma. Which of the following is the most appropriate first step in management?

a. Order a surgical consult immediately.

21. A 16-year-old arrives to your office soon after beginning basketball season. He states that he has had progressive pain in his knees. A physical examination reveals, in addition to tenderness, a swollen and prominent tibial tubercle. Radiographs of the area are unremarkable. Which of the following is the most likely diagnosis?

a. Osgood-Schlatter disease

279. At the 2-week checkup of a term female infant, the mother reports a grayish and sometimes bloody vaginal discharge since birth. The infant's mother and grandmother are the only caretakers. Examination of the external genitalia reveals an intact hymen with a thin grayish mucous discharge. Which of the following is the most appropriate next step?

a. Parental reassurance

236. A 10-year-old boy has been having "bellyaches" for about 2 years. They occur at night as well as during the day. Occasionally, he vomits after the onset of pain. Occult blood has been found in his stool. His father also gets frequent, nonspecific stomachaches. Which of the following is the most likely diagnosis?

a. Peptic ulcer

359. A 14-month-old infant suddenly develops a fever of 40.2°C (104.4°F). Physical examination shows an alert, active infant who drinks milk eagerly. No physical abnormalities are noted. The WBC count is 22,000/μL with 78% polymorphonuclear leukocytes, 18% of which are band forms. Which of the following is the most likely diagnosis?

a. Pneumococcal bacteremia

127. A 19-year-old primiparous woman develops toxemia in her last trimester of pregnancy and during the course of her labor is treated with magnesium sulfate. At 38 weeks' gestation, she delivers a 2100-g (4-lb, 10-oz) infant with Apgar scores of 1 at 1 minute and 5 at 5 minutes. Laboratory studies at 18 hours of age reveal a hematocrit of 79%, platelet count of 100,000/μL, glucose 41 mg/dL, magnesium 2.5 mEq/L, and calcium 8.7 mg/dL. Soon after, the infant has a generalized convulsion. Which of the following is the most likely cause of the infant's seizure?

a. Polycythemia

319. An irritable 6-year-old child has a somewhat unsteady but nonspecific gait. Physical examination reveals a very mild left facial weakness, brisk stretch reflexes in all four extremities, bilateral extensor plantar responses (Babinski reflex), and mild hypertonicity of the left upper and lower extremities; there is no muscular weakness. Which of the following is the most likely diagnosis?

a. Pontine glioma

453. A 6-day-old infant with severe hypotonia and poor feeding since birth

a. Prader-Willi syndrome

218. You have just given a 10-year-old boy an injection of pollen extract as prescribed by his allergist. You are about to move on to the next patient when the boy starts to complain about nausea and a funny feeling in his chest. You note that his face is flushed and his voice sounds muffled and strained. Which of the following is the first priority in managing this episode of anaphylaxis?

a. Preparation for endotracheal intubation

287. A 5-year-old girl without past history of UTI is in the hospital on antibiotics for Escherichia coli pyelonephritis. She is still febrile after 4 days of appropriate antibiotics. A renal ultrasound revealed no abscess, but a focal enlargement of one of the lobes of the right kidney. CT of the abdomen reveals a wedgeshaped area in the right kidney distinct from the normal tissue with minimal contrast enhancement. Appropriate management of this patient includes which of the following interventions?

a. Prolonged antibiotic therapy

139. You are called to a delivery of a term infant, about to be born via cesarean section to a mother with multiple medical problems, including a 1-month history of a seizure disorder, for which she takes phenytoin; rheumatic heart disease, for which she must take penicillin daily for life; hypertension, for which she takes propranolol; acid reflux, for which she takes aluminum hydroxide; and a deep venous thrombosis in her left calf diagnosed 2 days ago, for which she was started on a heparin infusion. The obstetrician is concerned about the possible effects of the mother's multiple medications on the newborn infant. Which of the following medications is most likely to cause harm in this newborn infant at delivery?

a. Propranolol

364. A 14-year-old girl awakens with a mild sore throat, low-grade fever, and a diffuse maculopapular rash. During the next 24 hours, she develops tender swelling of her wrists and redness of her eyes. In addition, her physician notes mild tenderness and marked swelling of her posterior cervical and occipital lymph nodes. Four days after the onset of her illness, the rash has vanished. Which of the following is the most likely diagnosis?

a. Rubella

178. The parents of a 2-month-old baby boy are concerned about his risk of coronary artery disease because of the recent death of his 40-year-old maternal uncle from a myocardial infarction. Which of the following is the most appropriate management in this situation?

a. Screen the parents for total cholesterol.

46. The obstetrical resident on call asks you to evaluate an area of a newborn's scalp that seems to have no hair and is scaly and yellowish

a. Sebaceous nevus

244. An 8-year-old is accidentally hit in the abdomen by a baseball bat. After several minutes of discomfort, he seems to be fine. Over the ensuing 24 hours, however, he develops a fever, abdominal pain radiating to the back, and persistent vomiting. On examination, the child appears quite uncomfortable. The abdomen is tender, with decreased bowel sounds throughout, but especially painful in the midepigastric region with guarding. Which of the following tests is most likely to confirm the diagnosis?

a. Serum amylase levels

256. You are admitting to the hospital a 3-month-old infant who has been having poor feeding, emesis, and diarrhea for 3 days. In the emergency center, her electrolytes were found to be: sodium 157 mEq/L, potassium 2.6 mEq/L, chloride 120 mEq/L, bicarbonate 14 mEq/L, creatinine 1.8 mEq/L, blood urea nitrogen (BUN) 68 mEq/L, and glucose 195 mEq/L. She was given a fluid bolus in the emergency center and has subsequently produced urine. Which of the following is the most appropriate next step in her management?

a. Slow rehydration over 48 hours

165. Phenylketonuria

a. Special infant formula

Questions 164 to 167 Blood samples of a 3-day-old full-term infant are sent for screening to identify diseases that would have serious, permanent consequences without prompt and appropriate treatment. Select the most appropriate treatment for each disease below. Each lettered option may be used once, more than once, or not at all. 164. Galactosemia

a. Special infant formula

324. A 9-year-old child has developed headaches that are more frequent in the morning and are followed by vomiting. Over the previous few months, his family has noted a change in his behavior (generally more irritable than usual) and his school performance has begun to drop. Imaging of this child is most likely to reveal a lesion in which of the following regions?

a. Subtentorial

40. As part of your anticipatory guidance to new parents of a healthy newborn, you suggest putting the child in which of the following positions for sleep?

a. Supine position

113. A mother delivers a neonate with meconium staining and Apgar scores of 3 at 1 and 5 minutes of life. She had no prenatal care and the delivery was by emergency cesarean section for what the obstetricians report as "severe fetal bradycardia." Which of the following sequelae could be expected to develop in this intubated neonate with respiratory distress?

a. Sustained rise in pulmonary arterial pressure

337. A 12-year-old boy has chronic headache that worsens during the school day. These headaches are not associated with nausea or emesis, and he does not have any symptoms prior to the headache.

a. Tension headache

114. A 2-year-old boy is being followed for congenital cytomegalovirus (CMV) infection. He is deaf and developmentally delayed. The child's mother informs you that she has just become pregnant and is concerned that the new baby will be infected and may develop serious consequences. Which of the following is true?

a. The mother has antibodies to CMV that are passed to the fetus

122. A 3-day-old infant, born at 32 weeks' gestation and weighing 1700 g (3 lb, 12 oz), has three episodes of apnea, each lasting 20 to 25 seconds and occurring after a feeding. During these episodes, the heart rate drops from 140 to 100 beats per minute, and the child remains motionless; between episodes, however, the child displays normal activity. Blood sugar is 50 mg/dL and serum calcium is normal. Which of the following is most likely true regarding the child's apneic periods?

a. They are due to an immature respiratory center

126. You are examining a newborn infant in the well-baby nursery. The infant was the product of a benign pregnancy and vaginal delivery; he appears to be in no distress. Interestingly, your measurement of frontooccipital head circumference is about 2 cm larger than the initial measurement done several hours before. Your examination otherwise is significant for tachycardia and a "squishy" feel to the entire scalp. You can elicit a fluid wave over the scalp. Management of this condition should include which of the following?

a. Transfer to the newborn ICU

Questions 338 to 340 338. Eye blinking or throat-clearing noises in an otherwise healthy 8-yearold boy.

a. Transient tic disorder of childhood

28. A mother arrives to the clinic with her three children (ages 2 months, 18 months, and 36 months). The 18-month-old has an intensely pruritic scalp, especially in the occipital region, with 0.5-mm lesions noted at the base of hair shafts, as shown in the picture. Which of the following therapies should be avoided in this situation?

a. Treatment of all household contacts with 1% lindane (Kwell)

Questions 63 to 66 63. A newborn infant is noted to have microcephaly with sloping forehead, cutis aplasia on the scalp, microphthalmia, and cleft lip and palate. His echocardiogram demonstrates a complex heart lesion including atrial septal defect (ASD), ventricular septal defect (VSD), and dextrocardia.

a. Trisomy 13

465. A 6-year-old girl with cognitive delay, a blowing systolic heart murmur, short stature, round face, bilateral transpalmar crease, upslanting palpebral fissures, small ears, and epicanthal folds.

a. Trisomy 21 (Down syndrome)

374. A 2-month-old infant comes to the emergency center with fever for 2 days, emesis, a petechial rash, and increasing lethargy. In the ambulance he had a 3-minute generalized tonic/clonic seizure that was aborted with lorazepam. He does not respond when blood is drawn or when an IV is placed, but he continues to ooze blood from the skin puncture sites. On examination, his anterior fontanelle is open and bulging. His CBC shows a WBC of 30,000 cells/μL with 20% band forms. Which of the infant's problems listed below is a contraindication to lumbar puncture?

a. Uncorrected bleeding diathesis

273. A 4-year-old whose diet consists mostly of cheese puffs and cola begins to have problems walking at night, complaining that he cannot see well. In addition, his skin has become dry and scaly, and he has complained of headache for a month.

a. Vitamin A

406. On a routine well-child examination, a 1-year-old boy is noted to be pale. He is in the 75th percentile for weight and the 25th percentile for length. Results of physical examination are otherwise normal. His hematocrit is 24%. The answer to which of the following questions is most likely to be helpful in making a diagnosis?

a. What is the child's usual daily diet?

181. A newborn is diagnosed with congenital heart disease. You counsel the family that the incidence of heart disease in future children is which of the following?

b. 2% to 6%

277. A newborn with arching of the back temporally related to feeds but no emesis

b. 24-hour pH probe

230. Perform thoracentesis to remove air under pressure.

b. 7.20 70 41 2

233. Perform tonsillectomy.

b. 7.36 60 50 7

83. Pyridoxine

b. A 6-year-old who has developed ataxia and sensory neuropathy.

241. A 3-year-old child presents to your office for an evaluation of constipation. The mother notes that since birth, and despite frequent use of stool softeners, the child has only about one stool per week. He does not have fecal soiling or diarrhea. He was born at term and without pregnancy complications. The child stayed an extra day in the hospital at birth because he did not pass stool for 48 hours, but has not been in the hospital since. Initial evaluation of this child should include which of the following?

b. A barium enema and rectal manometry

210. You admitted to the hospital the previous evening a 1-year-old boy who presented with cough, fever, and mild hypoxia. At the time of his admission, he had evidence of a right upper lobe consolidation on his chest radiograph. A blood culture has become positive in less than 24 hours for Staphylococcus aureus. Approximately 20 hours into his hospitalization, the nurse calls you because the child has acutely worsened over the previous few minutes, with markedly increased work in breathing, increasing oxygen requirement, and hypotension. As you move swiftly to the child's hospital room, you tell the nurse to order which of the following?

b. A large-bore needle and chest tube kit for aspiration of a probable tension pneumothorax

118. A primiparous woman whose blood type is O positive gives birth at term to an infant who has A-positive blood and a hematocrit of 55%. A total serum bilirubin level obtained at 36 hours of age is 12 mg/dL. Which of the following additional laboratory findings would be characteristic of ABO hemolytic disease in this infant?

b. A positive direct Coombs test

Questions 194 to 196 194. An infant in the neonatal intensive care unit (NICU) has shortened lower distal arms but with thumbs. Her platelet count is profoundly low.

b. ASD

308. The 7-year-old boy now in your office was last seen 2 weeks ago with a mild viral upper respiratory tract infection. Today, however, he presents with fever, ataxia, weakness, headache, and emesis. In the office he has a 3 minute left-sided tonic-clonic seizure. You send him to the hospital and order a magnetic resonance imaging (MRI) of the brain, the results of which show disseminated multifocal white matter lesions that enhance with contrast. This boy's likely diagnosis is which of the following?

b. Acute disseminated encephalomyelitis

128. An infant who appears to be of normal size is noted to be lethargic and somewhat limp after birth. The mother is 28 years old, and this is her fourth delivery. The pregnancy was uncomplicated, with normal fetal monitoring prior to delivery. Labor was rapid, with local anesthesia and intravenous meperidine (Demerol) administered for maternal pain control. Which of the following therapeutic maneuvers is likely to improve this infant's condition most rapidly?

b. Administration of naloxone (Narcan)

8. A mother calls you on the telephone and says that your 4-year-old son bit the hand of her 2-year-old son 48 hours previously. The area around the injury has become red, indurated, and swollen, and he has a temperature of 39.4°C (103°F). Which of the following is the most appropriate response?

b. Admit the child to the hospital immediately for surgical debridement and antibiotic treatment.

115. A full-term infant is born after a normal pregnancy; delivery, however, is complicated by marginal placental separation. At 12 hours of age, the child, although appearing to be in good health, passes a bloody meconium stool. For determining the cause of the bleeding, which of the following diagnostic procedures should be performed first?

b. An Apt test

343. Tuberous sclerosis

b. An infant with infantile spasms, a hypsarrhythmic EEG pattern, and ash-leaf depigmentation on her back

176. A 10-year-old boy, the star pitcher for the Salt Lake City Little League baseball team, had a sore throat about 2 weeks ago but did not tell anyone because he was afraid he would miss the play-offs. Since several children have been diagnosed with rheumatic fever in the area, his mother is worried that he may be at risk as well. You tell her that several criteria must be met to make the diagnosis but the most common finding is which of the following?

b. Arthralgia

297. A 7-year-old boy has cramping abdominal pain and a rash mainly on the back of his legs and buttocks as well as on the extensor surfaces of his forearms. Laboratory analysis reveals proteinuria and microhematuria. You diagnose Henoch-Schönlein, or anaphylactoid, purpura. In addition to his rash and abdominal pain, what other finding is he likely to have?

b. Arthritis or arthralgia

321. A 6-year-old child is hospitalized for observation because of a short period of unconsciousness after a fall from a playground swing. He has developed unilateral pupillary dilatation, focal seizures, recurrence of depressed consciousness, and hemiplegia. Which of the following is the most appropriate management at this time?

b. CT scan

252. A 6-week-old infant is admitted to the hospital with jaundice. Her outpatient blood work demonstrated a total bilirubin of 12 mg/dL with a direct portion of 3.5 mg/dL. Which of the following disorders is most likely to be responsible?

b. Choledochal cyst

362. The parents of a 3-year-old patient followed in your clinic recently took their child on quickly planned 5-day trip to Africa to visit an ill grandparent. Everyone did well on the trip, but since their return about 10 days ago the boy has been having intermittent, spiking fevers associated with headache, sweating, and nausea. The parents had not been too concerned since he was relatively well, except for being tired, between the fevers. Today, however, they feel that he looks a bit pale and his eyes appear "yellow." Which of the following is likely to reveal the source of his problem?

b. Complete blood count (CBC) with smear

438. A 7-day-old boy is admitted to a hospital for evaluation of vomiting and dehydration. Physical examination is otherwise normal except for minimal hyperpigmentation of the nipples. Serum sodium and potassium concentrations are 120 mEq/L and 9 mEq/L (without hemolysis), respectively; serum glucose is 40 mg/dL. Which of the following is the most likely diagnosis?

b. Congenital adrenal hyperplasia

150. A previously healthy full-term infant has several episodes of duskiness and apnea during the second day of life. Diagnostic considerations should include which of the following?

b. Congenital heart disease

227. A previously healthy 2-year-old black child has developed a chronic cough during the previous 6 weeks. He has been seen in different emergency rooms on two occasions during this period and has been placed on antibiotics for pneumonia. Upon auscultation, you hear normal breath sounds on the left. On the right side, you hear decreased air movement during inspiration but none upon expiration. Inspiratory (A) and expiratory (B) radiographs of the chest are shown below. Which of the following is the most appropriate next step in making the diagnosis in this patient?

b. Consult pediatric surgery for bronchoscopy

33. You are seeing an established patient, a 4-year-old girl brought in by her mother for vaginal itching and irritation. She is toilet trained and has not complained of frequency or urgency, nor has she noted any blood in her urine. Her mother noted she has been afebrile and has not complained of abdominal pain. Mom denies the risk of inappropriate contact; the girl also denies anyone "touching her there." Your physical examination of the perineum is significant for the lack of foul odor or discharge. You do note some erythema of the vulvar area but no evidence of trauma. Which of the following is the most appropriate course of action?

b. Counsel mother to stop giving the girl bubble baths, have the girl wear only cotton underwear, and improve hygiene.

65. A 6-week-old boy was small for his birth weight and had intrauterine growth retardation. He is microcephalic, has a rounded face, hypertelorism, and epicanthal folds. His cry is high-pitched

b. Cri du chat syndrome

399. Your sister who lives in another state sends via e-mail photographs of her 6-month-old infant. You note the child has a white reflection from one of his eyes. You hastily assist in arranging an urgent pediatric ophthalmologic evaluation. Your sister immediately accesses the Internet and begins to ask questions of you. Which of the following statements found by your sister is correct?

b. Cure rates for retinoblastoma treated in the United States exceed 90%.

361. The parents of a 7-day-old infant bring her to your office for a swollen eye. Her temperature has been normal, but for the last 2 days she has had progressive erythema and swelling over the medial aspect of the right lower lid near the punctum. Her sclera and conjunctiva are clear. Gentle pressure extrudes a whitish material from the punctum. Which of the following ophthalmic conditions is the correct diagnosis?

b. Dacryocystitis

131. After an uneventful labor and delivery, an infant is born at 32 weeks' gestation weighing 1500 g (3 lb, 5 oz). Respiratory difficulty develops immediately after birth and increases in intensity thereafter. At 6 hours of age, the child's respiratory rate is 60 breaths per minute. Examination reveals grunting, intercostal retraction, nasal flaring, and marked cyanosis in room air. Auscultation reveals poor air movement. Physiologic abnormalities compatible with these data include which of the following?

b. Decreased lung compliance, reduced lung volume, right-to-left shunt of blood

326. At birth, an infant is noted to have an abnormal neurologic examination. Over the next few weeks he develops severe progressive central nervous system (CNS) degeneration, an enlarged liver and spleen, macroglossia, coarse facial features, and a cherry-red spot in the eye. Which of the following laboratory findings most likely explains this child's problem?

b. Deficient activity of acid β-galactosidase

Questions 76 to 78 76. A 14-year-old male presents after taking a "happy pill" that his friend gave him. He is alert and oriented, but complains of a muscle spasm in his neck, making his head lean on his right shoulder. You also notice he is arching his back in an unusual manner.

b. Diphenhydramine (Benadryl)

491. The parents of a 16-year-old girl complain that she does not get enough sleep. They recently discovered that she stays awake most nights until 1:00 AM reading and text messaging her friends. She wakes at 6:30 AM for school, and complains of sleepiness during the day. On weekends she sleeps until noon. Her parents have tried taking away her computer and phone, but she still would go to bed at the same time. The parents are looking for advice in dealing with their "night owl" daughter. Which of the following is appropriate advice for this family?

b. Effects of puberty on melatonin cause a phase delay with later sleep onset.

262. An 18-month-old infant is found with the contents of a bottle of drain cleaner in his mouth. Which of the following treatment options is most appropriate?

b. Endoscopic examination within the first 12 to 24 hours

10. A chubby 6-month-old baby boy is brought to the clinic by his father. His father is concerned that his penis is too small (see photograph). The child is at the 95% for weight and the 50% for length; he has been developing normally and has had no medical problems. Which of the following is the most appropriate first step in management of this child?

b. Evaluation of penile length after retracting the skin and fat lateral to the penile shaft

148. A well-appearing, 3200-g (7-lb, 1-oz) black infant is noted to have fifth finger (postaxial) polydactyly. The extra digit has no skeletal duplications and is attached to the rest of the hand by a threadlike soft tissue pedicle (see photograph). Appropriate treatment for this condition includes which of the following?

b. Excision of extra digit

266. A 2-year-old has crampy abdominal pain and grossly bloody diarrheal stool, but no fever. His abdominal examination reveals no masses. A family member, who ate at the same local hamburger shop the night prior, has the same symptoms.

b. Fiberoptic endoscopy

268. A 10-year-old has complained for 1 month of intermittent epigastric pain that awakens him from sleep. He notes that eating food sometimes helps. He reports black stools during the prior week, and also admits that he has occasionally vomited frank blood.

b. Fiberoptic endoscopy

58. A skier recently rescued from a snowbank following an avalanche (caused by a barking Chihuahua) complains about his feet. Upon rescue they were whitish yellow and numb, but now they are blotchy and painful.

b. Frostbite

281. A 17-year-old boy is brought to the emergency department by his parents with the complaint of coughing up blood. He is stabilized, and his hemoglobin and hematocrit levels are 11 mg/dL and 33%, respectively. During his hospitalization, he is noted to have systolic blood pressure persistently greater than 130 mm Hg and diastolic blood pressure greater than 90 mm Hg. His urinalysis is remarkable for hematuria and proteinuria. You are suspicious the patient has which of the following?

b. Goodpasture syndrome

288. A 4-year-old boy, whose past medical history is positive for three urinary tract infections, presents with a blood pressure of 135/90 mm Hg. He is likely to exhibit which of the following symptoms or signs?

b. Headache

418. An infant is born to a woman who has received very little prenatal care. The mother is anxious, complains of heat intolerance and fatigue, and reports that she has not gained much weight despite having an increased appetite. On examination the mother is tachycardic, has a tremor, and has fullness in her neck and in her eyes. The infant is most likely at risk for development of which of the following?

b. Heart failure

134. You have been recently named as the medical director of the normal newborn nursery in your community hospital and have been asked to write standardized admission orders for all pediatricians to follow. Which of the following vaccines will you include on these orders?

b. Hepatitis B vaccine

405. A male infant was found to be jaundiced 12 hours after birth. At 36 hours of age, his serum bilirubin was 18 mg/dL, hemoglobin concentration was 12.5 g/dL, and reticulocyte count 9%. Many nucleated RBCs and some spherocytes were seen in the peripheral blood smear. The differential diagnosis should include which of the following?

b. Hereditary spherocytosis

310. A 6-month-old child was noted to be normal at birth, but over the ensuing months you have been somewhat concerned about his slowish weight gain and his mild delay in achieving developmental milestones. The family calls you urgently at 7:00 AM noting that their child seems unable to move the right side of his body. Which of the following conditions might explain this child's condition?

b. Homocystinuria

167. Hypothyroidism

b. Hormone therapy

251. A 15-year-old otherwise healthy boy presents with a complaint of intermittent abdominal distention, crampy abdominal pain, and excessive flatulence. He first started noticing these symptoms when he moved into his father's house, and his stepmother insisted on milk at dinner every night. He has normal growth, has not lost weight, and has no travel history. Which of the following is the most appropriate study to diagnose his condition?

b. Hydrogen excretion in breath after oral administration of lactose

Questions 197 to 200 197. A 15-year-old adolescent male presents to the office for a sports physical. In his screening questionnaire, he notes that he occasionally gets short of breath and dizzy during exercise, with occasional chest pain. He lost consciousness once last summer during football practice, but attributed it to the heat. His grandfather died suddenly at the age of 35 of unknown etiology, but otherwise the family is healthy.

b. Hypertrophic cardiomyopathy

117. A recovering premature infant who weighs 950 g (2 lb, 1 oz) is fed breast milk to provide 120 cal/kg/d. Over the ensuing weeks, the baby is most apt to develop which of the following?

b. Hypocalcemia

155. A newborn infant becomes markedly jaundiced on the second day of life, and a faint petechial eruption, first noted at birth, is now a generalized purpuric rash. Hematologic studies for hemolytic diseases are negative. Acute management should include which of the following?

b. Isolation of the infant from pregnant hospital personnel

142. The mother and father of a newborn come in for the 2-week checkup. The mother complains of "colic" and asks if she can switch to goat's milk instead of breast milk. Which of the following should be your main concern about using goat's milk instead of breast milk or cow's milk?

b. It has insufficient folate.

Questions 89 to 92 89. An 18-year-old friend of the family returns from spring break from a coastal town in Central America. He has an intensely pruritic lesion on his foot. The lesion is raised, red, serpiginous, and has a few associated bullae.

b. Ivermectin

189. A child has a 2-week history of spiking fevers, which have been as high as 40°C (104°F). She has spindle-shaped swelling of finger joints and complains of upper sternal pain. When she has fever, the parents note a faint salmon-colored rash that resolves with the resolution of the fever. She has had no conjunctivitis or mucositis, but her heart sounds are muffled and she has increased pulsus paradoxus. Which of the following is the most likely diagnosis?

b. Juvenile rheumatoid arthritis

454. A 5-year-old boy with mental retardation and polydactyly

b. Laurence-Moon-Biedl syndrome

457. A 14-year-old boy with hypogonadism and night blindness with retinitis pigmentosa

b. Laurence-Moon-Biedl syndrome

32. During the examination of a 2-month-old infant, you note that the infant's umbilical cord is still firmly attached. This finding prompts you to suspect which of the following?

b. Leukocyte adhesion deficiency

475. A 15-year-old female presents to your office with secondary amenorrhea. As part of your evaluation, you find that she is pregnant. After informing her of the pregnancy, you continue to explain that young mothers have a higher risk of several pregnancy-related complications, including which of the following?

b. Low-birthweight infants

214. A 10-year-old girl has had a "cold" for 14 days. In the 2 days prior to the visit to your office, she has developed a fever of 39°C (102.2°F), purulent nasal discharge, facial pain, and a daytime cough. Examination of the nose after topical decongestants shows pus in the middle meatus. Which of the following is the most likely diagnosis?

b. Maxillary sinusitis

23. You are seeing a 2-year-old child, brought by his father for a well-child examination. In providing age-appropriate anticipatory guidance, you should tell him which of the following?

b. Milk should be switched from whole to skim or low fat.

474. A developmentally delayed 2-year-old has bilateral hypopigmented whorls on the upper extremities

b. Mosaicism

332. A 3-year-old boy's parents complain that their child has difficulty walking. The child rolled, sat, and first stood at essentially normal ages and first walked at 13 months of age. Over the past several months, however, the family has noticed an increased inward curvature of the lower spine as he walks and that his gait has become more "waddling" in nature. On examination, you confirm these findings and also notice that he has enlargement of his calves. Which of the following is the most likely diagnosis?

b. Muscular dystrophy

72. After a fight with her boyfriend, a 16-year-old girl took "some pills." At presentation she is alert and complains of emesis, diaphoresis, and malaise. Her initial liver function tests, obtained about 12-hour postingestion, are elevated. Repeat levels at 24-hours show markedly elevated aspartate aminotransferase (AST) and alanine aminotransferase (ALT), along with abnormal coagulation studies and an elevated bilirubin.

b. N-acetylcysteine (Mucomyst)

463. A 1-year-old girl with glucose-6-phosphatase deficiency (von Gierke disease)

b. Na+ 125, K+ 3.0

18. The mother of a 3-day-old infant brings her child to your office for an early follow-up visit. The mom notes that the child has been eating well, has had no temperature instability, and stools and urinates well. She notes that over the previous 3 days the child has had a progressive "rash" on the face as pictured here. Which of the following is the most likely diagnosis?

b. Neonatal acne

333. Your 6-year-old son awakens at 1:00 AM screaming. You note that he is hyperventilating, is tachycardic, and has dilated pupils. He cannot be consoled, does not respond, and is unaware of his environment. After a few minutes, he returns to normal sleep. He recalls nothing the following morning. Which of the following is the most likely diagnosis?

b. Night terrors

52. A 4-year-old boy occasionally wakes in the middle of the night crying. When his parents check on him, he seems visibly frightened and tells his parents that Chihuahuas were chasing him.

b. Nightmares

394. An 11-month-old African American boy has a hematocrit of 24% on a screening laboratory done at his well-child checkup. Further testing demonstrates: hemoglobin 7.8 g/dL; hematocrit 22.9%; leukocyte count 12,200/μL with 39% neutrophils, 6% bands, 55% lymphocytes; hypochromia on smear; free erythrocyte protoporphyrin (FEP) 114 μg/dL; lead level 6 μg/dL whole blood; platelet count 175,000/μL; reticulocyte count 0.2%; sickle-cell preparation negative; stool guaiac-negative; and mean corpuscular volume (MCV) 64 fL. Which of the following is the most appropriate recommendation?

b. Oral ferrous sulfate

25. You are called by a general practitioner to consult on a patient admitted to the hospital 4 days ago. The patient is a 7-month-old white boy with poor weight gain for the past 3 months, who has not gained weight in the hospital despite seemingly adequate nutrition. You take a detailed diet history from his foster mother, and the amounts of formula and baby food intake seem appropriate for age. Physical examination reveals an active, alert infant with a strong suck reflex who appears wasted. You note generalized lymphadenopathy with hepatomegaly. In addition, you find a severe case of oral candidiasis that apparently has been resistant to treatment. Which of the following is the most appropriate next step in the evaluation or treatment of this child?

b. Order human immunodeficiency virus (HIV) polymerase chain reaction (PCR). testing because this is likely the presentation of congenitally acquired HIV.

327. The parents of a 2-year-old bring her to the emergency center after she had a seizure. Although the parents report she was in a good state of health, the vital signs in the emergency center reveal a temperature of 39°C (102.2°F). She is now running around the room. Which part of the story would suggest the best outcome in this condition?

b. Otitis media on examination

477. A 16-year-old boy who is the backup quarterback for the local high school team is in your office complaining of worsening acne. For the last few months he has noted more acne and more oily hair. On his examination, you note gynecomastia and small testicular volume. He is SMR 5. Which of the following drugs of abuse is the likely explanation for all of his findings?

b. Oxandrolone

350. An 8-year-old sickle-cell patient arrives at the emergency room (ER) in respiratory distress. Over the previous several days, the child has become progressively tired and pale. The child's hemoglobin concentration in the ER is 3.1 mg/dL. Which of the following viruses commonly causes such a clinical picture?

b. Parvovirus B19

37. Aunt Mary is helping her family move to a new apartment. During the confusion, 3-year-old Jimmy is noted to become lethargic. The contents of Aunt Mary's purse are strewn about on the floor. In the ER, the lethargic Jimmy is found to have miosis, bradycardia, and hypotension. He develops apnea, respiratory depression, and has to be intubated. His condition would most likely benefit from which of the following therapies?

b. Pediatric intensive care unit (PICU) support and trial of naloxone

19. A 2-year-old child (A) presents with a 4-day history of a rash limited to the feet and ankles. The papular rash is both pruritic and erythematous. The 3-month-old sibling of this patient (B) has similar lesions also involving the head and neck. The most appropriate treatment for this condition includes which of the following?

b. Permethrin

140. Your older sister, her husband, their 2-day-old infant, and their pet Chihuahua arrive at your door. The parents of the child are concerned because the pediatrician noted the child was "yellow" and ordered some studies. They produce a wad of papers for you to review. Both the mother and baby have O-positive blood. The baby's direct serum bilirubin is 0.2 mg/dL, with a total serum bilirubin of 11.8 mg/dL. Urine bilirubin is positive. The infant's white blood cell count is 13,000/μL with a differential of 50% polymorphonuclear cells, 45% lymphocytes, and 5% monocytes. The hemoglobin is 17 g/dL, and the platelet count is 278,000/μL. Reticulocyte count is 1.5%. The peripheral smear does not show fragments or abnormal cell shapes. Which of the following is the most likely explanation for this infant's skin color?

b. Physiologic jaundice

9. The adolescent shown presents with a 14-day history of multiple oval lesions over her back. The rash began with a single lesion over the lower abdomen (A); the other lesions developed over the next days (B). These lesions are slightly pruritic. Which of the following is the most likely diagnosis?

b. Pityriasis rosea

392. An 8-year-old girl with fever, tachypnea, and lobar infiltrate. She has failed outpatient therapy of amoxicillin, has developed empyema, and has had to have chest tubes placed.

b. Pneumococcal pneumonia

345. A 2-month-old child of an HIV-positive mother is followed in your pediatric practice. Which of the following therapies should be considered for this child?

b. Prophylaxis against Pneumocystis jiroveci pneumonia (Pneumocystis carinii)

381. Foot puncture wound through a tennis shoe of an adolescent exploring a construction site

b. Pseudomonas

Questions 96 to 102 96. Isoniazid therapy in a pregnant teenager

b. Pyridoxine

124. The father of a 1-week-old infant comes to the office in a panic. He has just noticed on his child a right anterior shoulder mass that seems tender. The father is an osteosarcoma survivor and fears the child has the same malignancy. In reviewing the baby's discharge papers, you note the child was a term, appropriate-for-gestational-age vaginal delivery with a birth weight of 3200 g (7 lb, 1 oz). Apgar scores were 9 at 1 and 5 minutes. Your examination is significant for a large firm mass on the right clavicle; the rest of the examination is normal. Management of this problem should include which of the following?

b. Reassurance and supportive care

135. A 1-week-old black infant presents to you for the first time with a large, fairly well-defined, purple lesion over the buttocks bilaterally, as shown in the photograph. The lesion is not palpable, and it is not warm nor tender. The mother denies trauma and reports that the lesion has been present since birth. This otherwise well-appearing infant is growing and developing normally and appears normal upon physical examination. Which of the following is the most appropriate course of action in this infant?

b. Reassurance of the normalcy of the condition

302. A 21-year-old woman presents to the emergency room in active labor. She has had no prenatal care, but her last menstrual period was approximately 9 months prior. Her membranes are artificially ruptured, yielding no amniotic fluid. She delivers an 1800-g (4-lb) term infant who develops significant respiratory distress immediately at birth. The first chest radiograph on this infant demonstrates hypoplastic lungs. After this infant is stabilized, which of the following is the most appropriate next step for this infant?

b. Renal ultrasound

476. A 12-year-old boy has scant, long, slightly pigmented pubic hairs; slight enlargement of his penis, and a pink, textured, and enlarged scrotum. He is most likely at which sexual maturation rating (SMR, also called Tanner) stage?

b. SMR 2

44. An adolescent boy complains of a splotchy red rash on the nape of his neck, discovered when he had his head shaved for football season. The rash seems to become more prominent with exercise or emotion. His mother notes that he has had the rash since infancy, but that it became invisible as hair grew. He had a similar rash on his eyelids that resolved in the newborn period.

b. Salmon patch

170. An infant presents with a large, flat vascular malformation over the left face and scalp. The mother notes that her other child was born with a capillary hemangioma on his arm and asks if this is the same thing. You explain that this vascular malformation is different, and that you will want to monitor him for another condition.

b. Seizures

171. A new mother points out several hypopigmented oval macules over the child's trunk and extremities. She notes that these have been present since birth. She is concerned because she had a brother with the same spots whom she thinks had "growths" in his brain.

b. Seizures

446. A 1-day-old normal-appearing infant develops tetany and convulsions. He was born at 34 weeks' gestation with Apgar scores of 2 and 4 (at 1 and 5 minutes, respectively) to a woman whose pregnancy was complicated by diabetes mellitus and pregnancy-induced hypertension. Which of the following serum chemistry values is likely to be the explanation for his condition?

b. Serum calcium of 6.2 mg/dL

105. A 3-year-old has had repeated episodes of sinusitis and otitis media. He was recently admitted for osteomyelitis of his femur with Staphylococcus aureus. The family notes that while his first 4 or 5 months of life were normal, he has been persistently ill with multiple infections in the ensuing months. The mother notes that her brother had similar problems with infections and died at the age of 3 years from a "lung infection." Physical examination is significant for the absence of lymph nodes and tonsillar tissue.

b. Serum immunoglobulin levels

Questions 67 to 70 67. An afebrile, obese 14-year-old boy has developed pain at the right knee and a limp.

b. Slipped capital femoral epiphysis...

495. A 19-year-old female has a few small papules on her labia and perineum. The papules become pustular, eroded, and ulcerated over the next few days; at the same time, the patient develops painful, tender inguinal lymphadenopathy.

b. Special chocolate agar culture

5. A previously healthy 4-year-old child pictured below presents to the emergency room (ER) with a 2-day history of a brightly erythematous rash and temperature of 40°C (104°F). The exquisitely tender, generalized rash is worse in the flexural and perioral areas. The child is admitted and over the next day develops crusting and fissuring around the eyes, mouth, and nose. The desquamation of skin shown in the photograph occurs with gentle traction. Which of the following is the most likely diagnosis?

b. Staphylococcal scalded skin syndrome

358. A 16-year-old boy presents to the emergency center with a 2-day history of an abscess with spreading cellulitis. While in the emergency center, he develops a high fever, hypotension, and vomiting with diarrhea. On examination you note a diffuse erythematous macular rash, injected conjunctiva and oral mucosa, and a strawberry tongue. He is not as alert as when he first arrived. This rapidly progressive symptom constellation is likely caused by which of the following disease processes?

b. TSST-1-secreting S aureus

153. Parents bring a 5-day-old infant to your office. The mother is O negative and was Coombs positive at delivery. The term child weighed 3055 g (6 lb, 1 oz) at birth and had measured baseline hemoglobin of 16 g/dL and a total serum bilirubin of 3 mg/dL. He passed a black tarlike stool within the first 24 hours of life. He was discharged at 30 hours of life with a stable axillary temperature of 36.5°C (97.7°F). Today the infant's weight is 3000 g, his axillary temperature is 35°C (95°F), and he is jaundiced to the chest. Parents report frequent yellow, seedy stool. You redraw labs and find his hemoglobin is now 14 g/dL, and his total serum bilirubin is 13 mg/dL. The change in which of the following parameters is of most concern?

b. Temperature

331. A newborn infant has respiratory distress and trouble feeding in the nursery. The mother has no significant medical history, but the pregnancy was complicated by decreased fetal movement. On physical examination, you note that aside from shallow respirations and some twitching of the fingers and toes, the infant is not moving, and is very hypotonic. In the mouth there is pooled saliva and you note tongue fasciculations. Deep tendon reflexes are absent. Spinal fluid is normal. Appropriate statements about this condition include which of the following statements?

b. The condition is caused by the degeneration of anterior horn cells in the spinal cord.

152. A woman gives birth to twins at 38 weeks' gestation. The first twin weighs 2800 g (6 lb, 3 oz) and has a hematocrit of 70%; the second twin weighs 2100 g (4 lb, 10 oz) and has a hematocrit of 40%. Which of the following statements is correct?

b. The first twin is more likely to have hyperbilirubinemia and convulsions

339. A 6-year-old boy with eye twitching and echolalia.

b. Tourette syndrome

191. A 3-day-old infant with a single second heart sound has had progressively deepening cyanosis since birth but no respiratory distress. Chest radiography demonstrates no cardiomegaly and normal pulmonary vasculature. An ECG shows an axis of 120° and right ventricular prominence. Which of the following congenital cardiac malformations is most likely responsible for the cyanosis?

b. Transposition of the great vessels

469. A small newborn with a large ventricular septal defect (VSD), clenched hands, cleft palate, rounded heels, and a horseshoe kidney.

b. Trisomy 18 (Edwards syndrome)

318. Examination of the cerebrospinal fluid (CSF) of an 8-year-old, mildly febrile child with nuchal rigidity and intermittent stupor shows the following: WBCs 85/μL (all lymphocytes), negative Gram stain, protein 150 mg/dL, and glucose 15 mg/dL. A computed tomographic (CT) scan with contrast shows enhancement of the basal cisterns by the contrast material. Which of the following is the most likely diagnosis?

b. Tuberculous meningitis

490. A 16-year-old girl presents with lower abdominal pain and fever. On physical examination, a tender adnexal mass is felt. Further questioning in private reveals the following: she has a new sexual partner; her periods are irregular; she has a vaginal discharge. Which of the following is the most likely diagnosis?

b. Tubo-ovarian abscess

204. A 6-year-old girl presents with a 2-day history of cough and fever. At your office she has a temperature of 39.4°C (103°F), a respiratory rate of 45 breaths per minute, and decreased breath sounds on the left side. Her chest x-ray is shown below. Which of the following is the most appropriate initial treatment?

b. Vancomycin

Questions 269 to 274 269. A 15-year-old vegetarian being treated for tuberculosis develops peripheral neuropathy

b. Vitamin B6

402. While bathing her newly-received 2-year-old son, a foster mother feels a mass in his abdomen. A thorough medical evaluation of the child reveals aniridia, hypospadias, horseshoe kidney, and hemihypertrophy. Which of the following is the most likely diagnosis for this child?

b. Wilms tumor

368. The parents of a 7-month-old boy arrive in your office with the child and a stack of medical records for a second opinion. The boy first started having problems after his circumcision in the nursery when he had prolonged bleeding. Studies were sent at the time for hemophilia, but factor VIII and IX activity were normal. At 2 months he developed bloody diarrhea, which his doctor assumed was a milk protein allergy and changed him to soy; his parents note he still has occasional bloody diarrhea. He has seen a dermatologist several times for eczema, and he has been admitted to the hospital twice for pneumococcal bacteremia. During both admissions, the parents were told that the infant's platelet count was low, but they have yet to attend the hematology appointment arranged for them. The child's WBC count and differential were normal. Which of the following is the most likely diagnosis in this child?

b. Wiskott-Aldrich syndrome

396. On a routine-screening CBC, a 1-year-old is noted to have a microcytic anemia. A follow-up hemoglobin electrophoresis demonstrates an increased concentration of hemoglobin A2. The child is most likely to have which of the following?

b. β-Thalassemia trait

254. An awake, alert infant with a 2-day history of diarrhea presents with a depressed fontanelle, tachycardia, sunken eyes, and the loss of skin elasticity. Which of the following is the correct percentage of dehydration?

c. 5% to 9%

232. Discontinue diuretics, discontinue base, and increase KCI in IV fluids.

c. 7.50 46 76 11

Questions 228 to 230 228. Have patient rebreathe in a paper bag

c. 7.64 18 94 1

Questions 496 to 500 496. Patellar tendinitis and Osgood-Schlatter disease

c. A 17-year-old starting center for a high school basketball team

132. A term infant is born to a known HIV-positive mother. She has been taking antiretroviral medications for the weeks prior to the delivery of her infant. Routine management of the healthy infant should include which of the following?

c. A course of zidovudine for the infant

296. The mother of a 2-year-old girl reports that her daughter complains of burning when she urinates and that she has foul-smelling discharge from her vagina. She has some slight staining on the front of her underwear, but denies fever, nausea, vomiting, or other constitutional signs. The child does not attend day care, and she has demonstrated no change in behavior. The physical examination is normal with an intact hymen, but the child's vulva is reddened and with a malodorous scent noted. Her urinalysis and culture are normal. Management of this condition includes which of the following?

c. Advice to stop taking prolonged bubble baths

245. A 10-month-old baby boy, recently adopted from Guyana, has a 5-hour history of crying, with intermittent drawing up of his knees to his chest. On the way to the emergency room he passes a loose, bloody stool. He has had no vomiting and has refused his bottle since the crying began. Physical examination is noteworthy for an irritable infant whose abdomen is very difficult to examine because of constant crying. His temperature is 38.8°C (101.8°F). The rectal ampulla is empty, but there is some gross blood on the examining finger. Which of the following studies would be most helpful in the immediate management of this patient?

c. Air contrast enema

116. As you are about to step out of a newly delivered mother's room, she mentions that she wants to breast-feed her healthy infant, but that her obstetrician was concerned about one of the medicines she was taking. Which of the woman's medicines, listed below, is clearly contraindicated in breast-feeding?

c. Amphetamines for her attention deficit disorder

Questions 341 to 343 341. Neurofibromatosis type 1

c. An 18-year-old with a history of fractures and optic gliomas, who now has developed a malignant schwannoma

387. Culture on modified Thayer-Martin medium

c. An infant born at home 2 days ago who has profuse purulent drainage from both eyes, lid edema, and chemosis

64. A 17-year-old boy has an unusual gait, large mouth with tongue protrusion, hypopigmentation with blond hair and pale blue eyes, and unprovoked bursts of laughter

c. Angelman syndrome

Questions 264 to 268 264. A newborn infant, the product of an emergency cesarean section, is 24 hours old and has a grossly bloody stool. She looks well otherwise.

c. Apt test

306. A 12-month-old girl whose height and weight are less than the fifth percentile; she has had several bouts of constipation and two previous admissions for dehydration. She is again admitted for dehydration and is noted to have serum potassium of 2.7 mEq/L.

c. Bartter syndrome

Questions 84 to 88 84. A 15-year-old with severe acne on an oral preparation of retinoic acid.

c. Bilateral microtia or anotia

186. A 3-month-old infant is brought to your office for pallor and listlessness. Your physical examination reveals tachycardia that is constant and does not vary with crying. He has no hepatomegaly and the lungs are clear. His ECG is shown. Which of the following is the most appropriate initial management of this patient?

c. Carotid massage

Questions 54 to 58 54. A 6-year-old returns from playing all day in the snow with several erythematous, ulcerative lesions on his fingertips; he complains the lesions are painful and itchy.

c. Chilblain

Questions 389 to 392 389. A 6-week-old infant with tachypnea and history of eye discharge at 2 weeks of age.

c. Chlamydial pneumonia

488. An 18-year-old male college student is seen in the student health clinic for urinary frequency, dysuria, and urethral discharge. Which of the following is likely to explain his condition?

c. Chlamydial urethritis

436. A 15-year-old boy has been immobilized in a double hip spica cast for 6 weeks after having fractured his femur in a skiing accident. He has become depressed and listless during the past few days and has complained of nausea and constipation. He is found to have microscopic hematuria and a blood pressure of 150/100 mm Hg. Which of the following is the most appropriate course of action?

c. Collect urine for measurement of the calcium to creatinine ratio

Questions 156 to 159 156. A newborn has bilateral cataracts and microphthalmia, intrauterine growth retardation, hemorrhagic skin lesions scattered throughout the body, and a harsh systolic murmur heard at the left sternal border and radiating to the lung fields.

c. Congenital rubella

36. A 5-year-old boy is brought into the ER immediately after an unfortunate altercation with a neighbor's immunized Chihuahua that occurred while the child was attempting to dress the dog as a superhero. The fully immunized child has a small, irregular, superficial laceration on his right forearm that has stopped bleeding. His neuromuscular examination is completely normal, and his perfusion is intact. Management should include which of the following?

c. Copious irrigation

247. A 14-year-old girl has a 9-month history of diarrhea, abdominal pain (usually periumbilical and postprandial), fever, and weight loss. She has had several episodes of blood in her stools. Which of the following is the most likely diagnosis?

c. Crohn disease

282. A 1-year-old child presents with failure to thrive, frequent large voids of dilute urine, excessive thirst, and three episodes of dehydration not associated with vomiting or diarrhea. Over the years, other family members reportedly have had similar histories. Which of the following is the most likely diagnosis?

c. Diabetes insipidus

136. A newborn infant develops respiratory distress immediately after birth. His abdomen is scaphoid. No breath sounds are heard on the left side of his chest, but they are audible on the right. Immediate intubation is successful with little or no improvement in clinical status. Emergency chest x-ray is shown (A) along with an x-ray 2 hours later (B). Which of the following is the most likely explanation for this infant's condition?

c. Diaphragmatic hernia

Questions 71 to 75 71. Over the past several weeks, a 2-year-old girl has exhibited developmental regression, abnormal sleep patterns, anorexia, irritability, and decreased activity. These symptoms have progressed to acute encephalopathy with vomiting, ataxia, and variable consciousness. The family recently moved, and they are in the process of restoring the interior of their home.

c. Dimercaptosuccinic acid (DMSA, succimer)

407. A 10-year-old boy is admitted to the hospital because of bleeding. Pertinent laboratory findings include a platelet count of 50,000/μL, prothrombin time (PT) of 15 seconds (control 11.5 seconds), activated partial thromboplastin time (aPTT) of 51 seconds (control 36 seconds), thrombin time (TT) of 13.7 seconds (control 10.5 seconds), and factor VIII level of 14% (normal 38%-178%). Which of the following is the most likely cause of his bleeding?

c. Disseminated intravascular coagulation (DIC)

213. A 4-year-old boy was admitted to the hospital last night with the complaint of "difficulty breathing." He has no past history of lung infection, no recent travel, and no day-care exposure; he does, however, have an annoying tendency to eat dirt. In the emergency center he was noted to be wheezing and to have hepatomegaly. He is able to talk, relaying his concern about his 6-week-old Chihuahua being left alone at home. Laboratory studies revealed marked eosinophilia (60% eosinophils). Which of the following tests is most likely to produce a specific diagnosis?

c. ELISA for Toxocara

312. A 4-year-old child is observed to hold his eyelids open with his fingers and to close one eye periodically, especially in the evening. He has some trouble swallowing his food. He usually appears sad, although he laughs often enough. He can throw a ball, and he runs well. Which of the following is most likely to aid in the diagnosis?

c. Effect of a test dose of edrophonium

435. A 4-year-old child has mental retardation, shortness of stature, brachydactyly (especially of the fourth and fifth digits), and obesity with round facies and short neck. The child is followed by an ophthalmologist for subcapsular cataracts, and has previously been noted to have cutaneous and subcutaneous calcifications, as well as perivascular calcifications of the basal ganglia. This patient is most likely to have which of the following features?

c. Elevated concentrations of parathyroid hormone

426. The parents of a 1-month-old infant bring him to the emergency center in your local hospital for emesis and listlessness. Both of his parents wanted a natural birth, so he was born at home and has not yet been to see a physician. On examination, you find a dehydrated, listless, and irritable infant. Although you don't have a birth weight, the parents do not feel that he has gained much weight. He has significant jaundice. His abdominal examination is significant for both hepatomegaly and splenomegaly. Laboratory values include a total bilirubin of 15.8 mg/dL and a direct bilirubin of 5.5 mg/dL. His liver function tests are elevated and his serum glucose is 38 mg/dL. Serum ammonia is normal. A urinalysis is negative for glucose, but it has a "mouse-like" odor. These findings are consistent with which of the following conditions?

c. Galactosemia

472. A mentally retarded 4-year-old boy who was noted to be hypotonic at birth and had failure to thrive in infancy now has a tremendous appetite, obesity, hypogonadism, and small hands and feet

c. Genomic imprinting

20. An 8-hour-old infant develops increased respiratory distress, hypothermia, and hypotension. A complete blood count (CBC) demonstrates a white blood cell (WBC) count of 2500/μL with 80% bands. The chest radiograph is shown below. Which of the following is the most likely diagnosis?

c. Group B streptococcal pneumonia

357. A 16-day-old infant presents with fever, irritability, poor feeding, and a bulging fontanelle. Spinal fluid demonstrates gram-positive cocci. Which of the following is the most likely diagnosis?

c. Group B streptococci

328. About 12 days after a mild upper respiratory infection, a 12-year-old boy complains of weakness in his lower extremities. Over several days, the weakness progresses to include his trunk. On physical examination, he has the weakness described and no lower extremity deep tendon reflexes, muscle atrophy, or pain. Spinal fluid studies are notable for elevated protein only. Which of the following is the most likely diagnosis in this patient?

c. Guillain-Barré syndrome

416. A 6-month-old infant has been exclusively fed a commercially available infant formula. Upon introduction of fruit juices, however, the child develops jaundice, hepatomegaly, vomiting, lethargy, irritability, and seizures. Tests for urine-reducing substances are positive. Which of the following is likely to explain this child's condition?

c. Hereditary fructose intolerance

149. An infant born at 35 weeks' gestation to a mother with no prenatal care is noted to be jittery and irritable, and is having difficulty feeding. You note coarse tremors on examination. The nurses report a high-pitched cry and note several episodes of diarrhea and emesis. You suspect which of the following?

c. Heroin withdrawal syndrome

466. A newborn has low sloping shoulders, right hand attached at elbow with agenesis of the forearm, cardiac abnormalities, missing chest wall musculature, and a bifid thumb.

c. Holt-Oram syndrome

393. Two weeks after a viral syndrome, a 2-year-old child develops bruising and generalized petechiae, more prominent over the legs. No hepatosplenomegaly or lymph node enlargement is noted. The examination is otherwise unremarkable. Laboratory testing shows the patient to have a normal hemoglobin, hematocrit, and white blood cell (WBC) count and differential. The platelet count is 15,000/μL. Which of the following is the most likely diagnosis?

c. Idiopathic (immune) thrombocytopenic purpura (ITP)

369. A 10-year-old boy from the Connecticut coast is seen because of discomfort in his right knee. He had a large, annular, erythematous lesion on his back that disappeared 4 weeks prior to the present visit. His mother recalls pulling a small tick off his back. Which of the following is a correct statement about this child's likely illness?

c. In addition to skin and joint involvement, CNS and cardiac abnormalities may be present.

375. The mother of one of your regular patients calls your office. She reports that her daughter has a 3-day history of subjective fever, hoarseness, and a bad barking cough. You arrange for her to be seen in your office that morning. Upon seeing this child, you would expect to find which of the following?

c. Infection with parainfluenza virus

175. A 14-year-old girl, angry at her mother for taking away her MP3 player, takes an unknown quantity of a friend's pills. Within the first hour she is sleepy, but in the emergency center she develops a widened QRS complex on her electrocardiogram (ECG), hypotension, and right bundle branch block. The therapy you would initiate for this ingestion is which of the following?

c. Intensive care unit (ICU) admission, close monitoring, and possible Fab antibody fragments

365. A 4-year-old child presents in the clinic with an illness notable for swelling in front of and in back of the ear on the affected side, as well as altered taste sensation. Correct statements about this condition include which of the following?

c. Involvement of the central nervous system (CNS) may occur 10 days after the resolution of the swelling.

183. A 2-year-old boy is brought into the emergency room with a complaint of fever for 6 days and the development of a limp. On examination, he is found to have an erythematous macular exanthem over his body as shown in image A, ocular conjunctivitis, dry and cracked lips, a red throat, and cervical lymphadenopathy. There is a grade 2/6 vibratory systolic ejection murmur at the lower left sternal border. A white blood cell (WBC) count and differential show predominant neutrophils with increased platelets on smear. Later, he develops the findings as seen in image B. Which of the following is the most likely diagnosis?

c. Kawasaki disease

50. A 15-month-old toddler continues to wake up crying every night. Her parents give her a nighttime bottle, rock her, and sing to her to help her go back to sleep. Her parents are exhausted and ask you if she is having bad dreams.

c. Learned behavior

53. A 5-year-old child refuses to sleep in his bed, claiming there are monsters in his closet and that he has bad dreams. The parents allow him to sleep with them in their bed to avoid the otherwise inevitable screaming fit. The parents note that the child sleeps soundly, waking only at sunrise.

c. Learned behavior

125. A 1-day-old infant who was born by a difficult forceps delivery is alert and active. She does not move her left arm spontaneously or during a Moro reflex. Rather, she prefers to maintain it internally rotated by her side with the forearm extended and pronated. The rest of her physical examination is normal. This clinical scenario most likely indicates which of the following?

c. Left-sided Erb-Duchenne paralysis

307. A 6-year-old child has had repeated episodes of otitis media. She undergoes an uneventful surgical placement of pressure-equalization (PE) tubes. In the recovery room she develops a fever of 40°C (104°F), rigidity of her muscles, and metabolic and respiratory acidosis. Which of the following is the most likely explanation for her condition?

c. Malignant hyperthermia

196. A new patient to your practice, a happy 10-year-old girl has a history of hyperextensible skin and easy bruising. She often has sprains and joint dislocations when she exercises.

c. Mitral valve prolapse

351. A 10-year-old boy was healthy until about 10 days ago when he developed 7 days of fever, chills, severe muscle pain, pharyngitis, headache, scleral injection, photophobia, and cervical adenopathy. After 7 days of symptoms he seemed to get better, but yesterday he developed fever, nausea, emesis, headache and mild nuchal rigidity. Cerebrospinal fluid (CSF) shows 200 white blood cells (WBC) per microliter (all monocytes) and an elevated protein. Correct statements about this infection include which of the following?

c. Most cases are mild or subclinical

360. A 21-year-old woman has just delivered a term infant. She has had only one visit to her obstetrician, and that was at about 6 weeks of pregnancy. She provides her laboratory results from that visit. The delivered infant is microcephalic, has cataracts, a heart murmur, and hepatosplenomegaly. Your further evaluation of the child demonstrates thrombocytopenia, mild hemolytic anemia, and, on the echocardiogram, patent ductus arteriosus and peripheral pulmonary artery stenosis. Which of the following maternal laboratory tests done at 6 weeks gestation is likely to explain the findings in this child?

c. Negative rubella titer

Questions 103 to 107 103. A 1-year-old boy has been admitted three times in the past with abscess formation requiring incision and drainage. He is now admitted for surgical drainage of a hepatic abscess identified on ultrasound

c. Nitroblue tetrazolium (NBT) test

31. An 8-month-old infant arrives to the emergency department (ED) with a 2-day history of diarrhea and poor fluid intake. Your quick examination reveals a lethargic child; his heart rate is 180 beats per minute, his respiratory rate is 30 breaths per minute, and his blood pressure is low for age. He has poor skin turgor, 5-second capillary refill, and cool extremities. Which of the following fluids is most appropriate management for his condition?

c. Normal saline

255. A 9-month-old is brought to the emergency center by ambulance. The child had been having emesis and diarrhea with decreased urine output for several days, and the parents noted that she was hard to wake up this morning. Her weight is 9 kg, down from 11 kg the week prior at her 9-month checkup. You note her heart rate and blood pressure to be normal. She is lethargic, and her skin is noted to be "doughy." After confirming that her respiratory status is stable, you send electrolytes, which you expect to be abnormal. You start an IV. Which of the following is the best solution for an initial IV bolus?

c. Normal saline (154 mEq sodium/L)

110. A 2-hour-old full-term newborn infant is noted by the nursing staff to be having episodes of cyanosis and apnea. Per nursery protocol they place an oxygen saturation monitor on him. When they attempted to feed him, his oxygen levels drop into the 60s. When he is stimulated and cries, his oxygen levels increase into the 90s. Which of the following is the most important next step to quickly establish the diagnosis?

c. Passage of catheter into nose

198. A 15-year-old adolescent female comes to be evaluated for syncopal episodes. Her only other medical problem is congenital deafness. She notes through a signing interpreter that syncopal episodes happen during stressful or emotional situations and that they started only within the past year or so. Her mother and father are from the same small farming town and are second cousins.

c. Prolonged QT syndrome

7. A patient comes to your office for a hospital follow-up. You had sent him to the hospital 3 weeks earlier for persistent fevers but no other symptoms; he was diagnosed with endocarditis and is currently being treated appropriately. Advice to this family should now include which of the following?

c. Provide the child with antibiotic prophylaxis for dental procedures.

133. Initial examination of a full-term infant weighing less than 2500 g (5 lb, 8 oz) shows edema over the dorsum of her hands and feet. Which of the following findings would support a diagnosis of Turner syndrome?

c. Redundant skin folds at the nape of the neck

154. You are called to a delivery of a woman with no prenatal care; she is in active labor but has no history of amniotic rupture. The biophysical profile done in the emergency center revealed severe oligohydramnios. When you get this infant to the nursery, you should carefully evaluate him for which of the following?

c. Renal agenesis

433. A 12-year-old girl has a solitary thyroid nodule found on routine examination; she has no symptoms. Which of the following is the most appropriate next step for this patient?

c. Serum thyroid function tests

17. You are called to the ER to see one of your patients. The father of this 14-year-old mildly retarded child says that he found the child about 20 minutes ago in the neighbor's garden shed with an unknown substance in his mouth. The child first had a headache, but then became agitated and confused; while you are talking to the father in the ER the child begins to have a seizure and dysrhythmia on the cardiac monitor. The blood gas demonstrates a severe metabolic acidosis. Which of the following agents is most likely the culprit?

c. Sodium cyanide

301. A 9-year-old boy comes to the office for a pre-participation physical examination for summer camp. His parents report that he still has episodes of bed-wetting. The boy's father confides that he also had bed-wetting until he was 10. They are concerned about the bed-wetting, but they are more concerned about their son's upcoming week at summer camp and that the other boys may harass him for wetting the bed. Which of the following statements about nocturnal enuresis is correct?

c. Spontaneous cure rates are high regardless of therapy.

173. A 6-month-old comatose infant has multiple broken bones in various stages of healing, a bulging anterior fontanelle, and retinal hemorrhages.

c. Subdural hemorrhage

224. A 13-year-old boy has a 3-day history of low-grade fever, symptoms of upper respiratory infection, and a sore throat. A few hours before his presentation to the emergency room, he has an abrupt onset of high fever, difficulty swallowing, and poor handling of his secretions. He indicates that he has a marked worsening in the severity of his sore throat. His pharynx has a fluctuant bulge in the posterior wall. A soft tissue radiograph of his neck is shown. Which of the following is the most appropriate initial therapy for this patient?

c. Surgical consultation for incision and drainage under general anesthesia

315. A 5-month-old child was normal at birth, but the family reports that the child does not seem to look at them any longer. They also report the child seems to "startle" more easily than he had before. Testing of his white blood cells (WBCs) identifies the absence of β-hexosaminidase A activity, confirming the diagnosis of which of the following?

c. Tay-Sachs disease

284. The mother of a 2-year-old male child states that she has noticed white, cheeselike material arising from his foreskin and also that he cannot fully retract the foreskin behind the glans penis. Which of the following is the correct advice for this parent?

c. The child is normal.

42. A 5-month-old child with poor growth presents to the ER with generalized tonic-clonic seizure activity of about 30-minute duration that stops upon the administration of lorazepam. Which of the following historical bits of information gathered from the mother is most likely to lead to the correct diagnosis in this patient?

c. The mother has been diluting the infant's formula to make it last longer

366. A bat is found in the bedroom of a 4-year-old patient while the boy is sleeping. The family and the patient deny close contact with or bites from the bat. Which of the following is a correct statement regarding this situation?

c. The patient should be started on the rabies vaccine series.

485. The recent suicide of a well-known high school cheerleader in your community has generated an enormous amount of community concern and media coverage. A girl who was close friends with the deceased makes an appointment and comes in to your office to discuss the event with you. You ask, and she denies suicidal ideation, but she has many questions about suicide. Correct statements about adolescent suicide include which of the following?

c. Those who are successful have a history of a prior attempt or prior serious suicidal ideation.

190. A cyanotic newborn is suspected of having congenital heart disease. He has an increased left ventricular impulse and a holosystolic murmur along the left sternal border. The ECG shows left-axis deviation and left ventricular hypertrophy (LVH). Which of the following is the most likely diagnosis?

c. Tricuspid atresia

29. A 2-year-old boy has been vomiting intermittently for 3 weeks and has been irritable, listless, and anorectic. His use of language has regressed to speaking single words. In your evaluation of this patient, which of the following is the most reasonable diagnosis to consider?

c. Tuberculous meningitis

243. A 17-year-old adolescent female is 6 weeks postpartum. She presents to the emergency room with the complaints of increased jaundice, abdominal pain, nausea, vomiting, and fever. Her examination is remarkable for jaundice, pain of the right upper quadrant with guarding, and a clear chest. Chest radiographs appear normal. Which of the following tests is most likely to reveal the cause of this pain?

c. Ultrasound of the right upper quadrant

276. A 12-year-old who has several weeks of abdominal pain and black stools

c. Upper GI endoscopy

Questions 335 to 337 335. A 15-year-old girl has an acute, recurrent, pulsatile headache localized behind the eyes that tends to occur more frequently around menses. She has no symptoms that occur prior to the headache; her neurologic examination is normal.

c. Vascular headache (migraine)

95. A 3-day-old infant was born to a mother with active systemic lupus erythematosus (SLE)

c. Vitamin A

98. Measles in developing countries

c. Vitamin A

272. A 17-month-old toddler has been irritable over the past month. She now refuses to walk and seems to have tenderness in both of her legs. She has had a low-grade fever, and she has petechiae on her skin and mucous membranes. She has a small cut that has not healed well. Radiographs of the legs reveal generalized bony atrophy with epiphyseal separation.

c. Vitamin C

166. Biotinidase deficiency

c. Vitamin therapy

Questions 168 to 171 168. A child's left arm and leg seem bigger than those on the right. In addition, the child has aniridia. None of the family members have aniridia or hemihypertrophy, nor do they know of anyone else in the family with these conditions.

c. Wilms tumor

188. A 4-year-old girl is brought to the pediatrician's office. Her father reports that she suddenly became pale and stopped running while he had been playfully chasing her and her pet Chihuahua. After 30 minutes, she was no longer pale and wanted to resume the game. She has never had a previous episode and has never been cyanotic. Her physical examination was normal, as were her chest x-ray and echocardiogram. An ECG showed the pattern seen on the next page, which indicates which of the following?

c. Wolff-Parkinson-White syndrome

180. A 15-year-old girl with short stature, neck webbing, and sexual infantilism is found to have coarctation of the aorta. A chromosomal analysis likely would demonstrate which of the following?

c. XO karyotype

437. An adolescent with type 1 diabetes returns for a follow-up visit after his annual checkup last week. You note that his serum glucose is elevated, and his glycosylated hemoglobin (hemoglobin A1C) is 16.7%. This finding suggests poor control of his diabetes over at least which of the following time periods?

d. 2 months

3. A child is brought to your clinic for a routine examination. She can put on a T-shirt but requires a bit of help dressing otherwise. She can copy a circle well but has difficulty in copying a square. Her speech is understandable and she knows four colors. She balances proudly on each foot for 2 seconds but is unable to hold the stance for 5 seconds. Which of the following is the most likely age of this child?

d. 4 years

229. Administer FIO2 0.4.

d. 7.34 32 39 8

234. Repeat the test because of obvious laboratory error

d. 7.41 60 90 10

Questions 386 to 388 386. Culture on sorbitol MacConkey medium

d. A toddler with bloody diarrhea, anemia, and acute renal failure

486. A 16-year-old girl is in your office for a preparticipation sports examination. She plans to play soccer in the fall, and needs her form filled out. Which of the following history or physical examination findings is usually considered a contraindication to playing contact sports?

d. Absence of a single eye

304. A 6-year-old girl with a complaint of "dark urine"; she has a blood pressure of 120/80 mm Hg.

d. Acute glomerulonephritis

138. You are advised by the obstetrician that the mother of a baby she has delivered is a carrier of hepatitis B surface antigen (HBsAg-positive). Which of the following is the most appropriate action in managing this infant?

d. Administer hepatitis B immune globulin and hepatitis B vaccine to the infant.

Questions 79 to 83 Excess vitamin intake has been shown to have deleterious effects. Match the vitamin with the toxic effect. Each lettered option may be used once, more than once, or not at all.. 79. Vitamin A

d. An irritable 8-year-old child with headache, vomiting, alopecia, dry/itchy skin with desquamation of the palms and soles, hepatosplenomegaly, and swelling of the bones.

260. A 12-year-old girl was hit in the face by a baseball 15 minutes earlier and has had her mandibular incisors knocked out. Which of the following represents the best plan of action?

d. Avulsed teeth can be transported in the mouth of the parent or a cooperative patient

38. As a city public health officer, you have been charged with the task of screening high-risk children for lead poisoning. Which of the following is the best screen for this purpose?

d. Blood lead level

163. A preterm infant is now 7 weeks old. She was intubated for 2 weeks and was weaned off oxygen at 3 weeks of age. You are about to leave your office for Thanksgiving holiday when the emergency room calls to tell you she has new hypoxia, respiratory distress, wheezes, and runny nose. A chest radiograph reveals patchy infiltrates and hyperexpansion in both lung fields. The newborn's 2-year-old sibling has an upper respiratory infection.

d. Bronchiolitis

151. The signs and symptoms of meningitis in an infant can be different than those in an adult. Which of the following signs and symptoms of meningitis is more helpful in an adult patient than in a 4-month-old?

d. Brudzinski sign

320. A 2-year-old boy has been doing well despite his diagnosis of tetralogy of Fallot. He presented to an outside ER a few days ago with a complaint of an acute febrile illness for which he was started on a "pink antibiotic." His mother reports that for the past 12 hours or so he has had a headache and is more lethargic than normal. On your examination he seems to have a severe headache, nystagmus, and ataxia. Which of the following would be the most appropriate first test to order?

d. CT or MRI of the brain

316. The family of a 4-year-old boy has just moved into your area. The child was recently brought to the emergency department (ED) for an evaluation of abdominal pain. Although appendicitis was ruled out in the ED and the child's abdominal pain has resolved, the ED physician requested that the family follow up in your office to evaluate an incidental finding of an elevated creatine kinase. The family notes that he was a late walker (began walking independently at about 18 months of age), that he is more clumsy than their daughter was at the same age (especially when trying to hold onto small objects), and that he seems to be somewhat sluggish when he runs, climbs stairs, rises from the ground after he sits, and rides his tricycle. A thorough history and physical examination are likely to reveal which of the following?

d. Cataracts

410. An otherwise healthy 17-year-old complains of swollen glands in his neck and groin for the past 6 months and an increasing cough over the previous 2 weeks. He also reports some fevers, especially at night, and possibly some weight loss. On examination, you notice that he has nontender cervical, supraclavicular, axillary, and inguinal nodes, no hepatosplenomegaly, and otherwise looks to be fairly healthy. Which of the following would be the appropriate next step?

d. Chest radiograph

201. You are asked to evaluate an infant born vaginally 3 hours previously to a mother whose only pregnancy complication was poorly controlled gestational diabetes. The nursing staff noticed that the infant was breathing abnormally. On examination, you find that the infant is cyanotic, has irregular, labored breathing, and has decreased breath sounds on the right side. You also note decreased tone in the right arm. You provide oxygen and order a stat portable chest radiograph, which is normal. Which of the following studies is most likely to confirm your diagnosis?

d. Chest ultrasound

225. A 6-week-old child arrives with a complaint of "breathing fast" and a cough. On examination you note the child to have no temperature elevation, a respiratory rate of 65 breaths per minute, and her oxygen saturation to be 94%. Physical examination also is significant for rales and rhonchi. The past medical history for the child is positive for an eye discharge at 3 weeks of age, which was treated with a topical antibiotic drug. Which of the following organisms is the most likely cause of this child's condition?

d. Chlamydia trachomatis

13. A 6-year-old boy is often teased at school because he has stooled in his underwear almost daily for the last 3 months. He was toilet trained at 2 years of age without difficulty, but over the last 2 years he had developed ongoing constipation. His family is frustrated because they cannot believe him when he says "I didn't know I had to go." He is otherwise normal; school is going well, and his home life is stable. His only finding on examination is significant for stool in the rectal vault. The plain radiograph of his abdomen is shown. Initial management of this problem should include which of the following?

d. Clear fecal impaction and short-term stool softener use

249. A 5-month-old child regularly regurgitates a large portion of her feeds. A pH probe study showed significant periods of low esophageal pH. The child has normal growth and no other significant past medical history. Which of the following is the best management at this point?

d. Close observation only

56. A 9-year-old girl presents during summer break with an area of erythematous, firm, and slightly swollen skin at the corner of her mouth and extending to her cheek. The area is not tender.

d. Cold panniculitis

479. A 15-year-old girl is seen in your clinic with a sprained ankle, which occurred the previous day while she was exercising in her room. You realize that you have not seen her for quite some time, and begin to expand your examination beyond the ankle. You find relatively minimal swelling on her right ankle. She has dental decay, especially of anterior teeth and a swollen, reddened, irritated uvula. She seems to be somewhat hirsute on her arms and legs, but has thinning of her hair of the head. She has a resting heart rate of 60 beats per minute, and her oral temperature is 35.5°C (96°F). Further questioning suggests that she has developed secondary amenorrhea. Which of the following is the most appropriate next step in the management of this girl?

d. Comparison of current and past weights

159. A newborn has microcephaly, intracranial calcifications, hepatosplenomegaly, and marked hyperbilirubinemia and thrombocytopenia

d. Congenital cytomegalovirus

147. The infant in the following picture presents with hepatosplenomegaly, anemia, persistent rhinitis, and a maculopapular rash. Which of the following is the most likely diagnosis for this child?

d. Congenital syphilis

200. An infant previously diagnosed with a large muscular VSD comes to the office with complaints from the mother of fatigue and poor feeding over the past month. You note the child has not gained weight since the previous visit 2 months ago. The child is apathetic, tachypneic, and has wheezes and crackles on lung auscultation.

d. Congestive heart failure

427. A 2-month-old boy is admitted to the hospital for failure to thrive. You note him to have fatty stools, and consider cystic fibrosis in your differential diagnosis. You order pilocarpine iontophoresis, but the laboratory calls to say they could not collect enough sweat from the infant. Which of the following is another way to make the diagnosis of cystic fibrosis?

d. DNA testing for cystic fibrosis transmembrane regulator (CFTR) mutations

408. A 17-year-old adolescent comes to your office seeking help for "heavy" menses. Your review of systems also reveals weekly epistaxis. Her only significant past history includes a tonsillectomy at age 6 after which she required blood transfusion for excessive bleeding. Her family history includes several people who seem to bleed and bruise more easily than others. The patient's mother required a hysterectomy after child birth for excessive hemorrhage. You order a variety of laboratory tests. The patient has a hemoglobin of 6.5 mg/dL with an MCV of 60%; her platelet count is 350,000/μL. Her von Willebrand antigen and her von Willebrand factor (vWF) activity (ristocetin cofactor activity) are decreased. Her vWF is reported as normal but in decreased amounts. You have been unable to reach her to report the findings, but when she calls about 1 week later she reports she is having a mild to moderate nosebleed. You initiate therapy with which of the following?

d. Desmopressin (DDAVP)

484. A 16-year-old girl, accompanied by her mother, is in your office for a well-adolescent visit. The mother asks about drug and alcohol abuse. You explain that the warning signs of abuse include which of the following?

d. Deterioration in personal habits, hygiene, dress, grooming, speech patterns, and fluency of expression

470. A newborn with hypoglycemia, hypocalcemia, and hypoplastic lower extremities.

d. Diabetic embryopathy

123. You have an 11-day-old term infant in your office for a well-child visit. The mother notes that she received a letter that day from the state's Department of Health reporting that her child's newborn screen had come back abnormal, indicating possible galactosemia. Which of the following is the most appropriate management at this point?

d. Discontinue breast-feeding and initiate soy formula feedings.

421. Friends are considering adopting a "special needs" child from another country. The family has few details, but the information they have received so far suggests the 4-year-old child has had surgery for an endocardial cushion defect, is short for his age, and had a history of what sounds like surgically repaired duodenal atresia at birth. You are suspicious this child may have which of the following syndromes?

d. Down

141. At the time of delivery, a woman is noted to have a large volume of amniotic fluid. At 6 hours of age, her baby begins regurgitating small amounts of mucus and bile-stained fluid. Physical examination of the infant is normal, and an abdominal x-ray is obtained (see below). Which of the following is the most likely diagnosis of this infant's disorder?

d. Duodenal atresia

340. An 8-year-old hospitalized boy with unusual "spasms" of his neck and arms shortly after receiving Phenergan for nausea caused by his chemotherapy.

d. Dystonia

187. A 2-year-old child with minimal cyanosis has an S3 and S4 (a quadruple rhythm), a systolic murmur in the pulmonic area, and a middiastolic murmur along the lower left sternal border. An ECG shows right atrial hypertrophy and a ventricular block pattern in the right chest leads. Which of the following is the most likely diagnosis?

d. Ebstein anomaly

334. A previously healthy 16-year-old girl presents to the emergency center with the complaint of "falling out." She was with her friends at a local fast food restaurant when she felt faint and, according to her friends, lost consciousness for about a minute. There was no seizure activity noted, but the friends did notice her arms twitching irregularly. She is now acting normally. She denies chest pain or palpitations, and her electrocardiogram (ECG) is normal. Further management of this patient should include which of the following?

d. Encourage adequate fluid and salt intake

352. A previously healthy 8-year-old boy has a 3-week history of low-grade fever of unknown source, fatigue, weight loss, myalgia, and headaches. On repeated examinations during this time, he is found to have developed a heart murmur, petechiae, and mild splenomegaly. Which of the following is the most likely diagnosis?

d. Endocarditis

354. The 3-year-old sister of a newborn baby develops a cough diagnosed as pertussis by nasopharyngeal culture. The mother gives a history of having been immunized as a child. Which of the following is a correct statement regarding this clinical situation?

d. Erythromycin should be administered to the infant.

78. A 17-year-old is brought into the ED by his friends at about 10:00 AM. They were at a party the night before and drank some "homemade" alcohol. He is disoriented and confused, and has an anion-gap acidosis. He begins to have seizures

d. Ethanol

108. The term infant pictured below weighs 2200 g (4 lb, 14 oz). He is found to have a ventricular septal defect on cardiac evaluation. This infant appears to have features consistent with which of the following?

d. Fetal alcohol syndrome

137. Shortly after birth, an infant develops abdominal distention and begins to drool. When she is given her first feeding, it runs out the side of her mouth, and she coughs and chokes. Physical examination reveals tachypnea, intercostal retractions, and bilateral pulmonary rales. The esophageal anomaly that most commonly causes these signs and symptoms is illustrated by which of the following? (ישנו ציור שאני לא יכולתי להעתיק***)

d. Figure D

41. A mentally retarded 14-year-old boy has a long face, large ears, micropenis, and large testes. Chromosome analysis is likely to demonstrate which of the following?

d. Fragile X syndrome

246. A 12-month-old girl has been spitting up her meals since 1 month of age. Her growth is at the 95th percentile, and she is otherwise asymptomatic and without findings on physical examination. Which of the following is the most likely diagnosis?

d. Gastroesophageal reflux

417. A 12-year-old healthy girl has some dizziness at synagogue. In your office you find her to have a hemoglobin of 8 mg/dL, a white blood cell (WBC) count of 4000/μL, and a platelet count of 98,000/μL. Physical examination reveals an enlarged spleen. An x-ray of the femur is described as "appearing to be an Erlenmeyer flask." Bone marrow examination shows abnormal cells. The diagnosis can be confirmed by measurement of activity of which of the following?

d. Glucocerebrosidase

130. The newborn pictured below was born at home and has puffy, tense eyelids; red conjunctivae; a copious amount of purulent ocular discharge; and chemosis 2 days after birth. Which of the following is the most likely diagnosis?

d. Gonococcal ophthalmia

428. An otherwise healthy 7-year-old child is brought to you to be evaluated because he is the shortest child in his class. Careful measurements of his upper and lower body segments demonstrate normal body proportions for his age. Which of the following disorders of growth should remain in your differential?

d. Growth hormone deficiency

403. A healthy 1-year-old child comes to your office for a routine checkup and for immunizations. His parents have no complaints or concerns. The next day, the CBC you performed as customary screening for anemia returns with the percentage of eosinophils on the differential to be 30%. Which of the following is the most likely explanation?

d. Helminth infestation

283. A 6-month-old infant has poor weight gain, vomiting, episodic fevers, and chronic constipation. Laboratory studies reveal a urinalysis with a pH of 8.0, specific gravity of 1.010, 1+ glucose, and 1+ protein. Urine anion gap is normal. Serum chemistries show a normal glucose and a normal albumin with a hyperchloremic metabolic acidosis. Serum phosphorus and calcium are low. What is the best diagnosis to explain these findings?

d. Hereditary Fanconi syndrome

179. For the past year, a 12-year-old boy has had recurrent episodes of swelling of his hands and feet, which has been getting worse recently. These episodes occur following exercise and emotional stress, last for 2 to 3 days, and resolve spontaneously. The last episode was accompanied by abdominal pain, vomiting, and diarrhea. The results of routine laboratory workup are normal. An older sister and a maternal uncle have had similar episodes, but they were not given a diagnosis. He presents today with another episode as shown in the photographs on the next page. Which of the following is the most likely diagnosis?

d. Hereditary angioedema

383. The appearance of an evanescent, erythematous, maculopapular rash following the rapid defervescence of several days of high fever in a 9-monthold boy

d. Human herpesvirus 6

253. A 2-year-old arrives in the emergency center after having swallowed a button battery from one of her toys. She is breathing comfortably, without stridor. Radiographs show the battery to be lodged in the esophagus. Which of the following is the correct next step?

d. Immediate removal of the battery via endoscopy.

336. A 7-year-old boy has chronic, worsening headache without preceding symptoms. He complains of emesis in the morning before breakfast for the last 2 weeks.

d. Increased intracranial pressure

487. A 15-year-old boy is in the office for a preparticipation sports physical examination before he begins playing with the varsity football team at his school. Although he is a skilled receiver, he will be one of the smallest players on the field and is concerned about the potential for injury. He asks how to bulk up. Appropriate advice to increase muscle mass includes which of the following?

d. Increasing muscle work

215. You are awakened in the night by your 2-year-old son, who has developed noisy breathing on inspiration, marked retractions of the chest wall, flaring of the nostrils, and a barking cough. He has had a mild upper respiratory infection (URI) for 2 days. Which of the following therapies is indicated?

d. Inhaled epinephrine and a dose of steroids

39. A 15-year-old is participating in high school football practice in August in Texas. He had complained of headache and nausea earlier in practice, but kept playing after a cup of water. He is now confused and combative. He is dizzy and sweating profusely. His temperature is 41°C (105.8°F). Therapy should consist of which of the following?

d. Initiate whole body cold water immersion

290. A 6-week-old child is being evaluated for a fever of unknown etiology. As part of the laboratory evaluation, a urine specimen was obtained that grew E coli with a colony count of 2000/μL. These findings would be definite evidence of a urinary tract infection if which of the following is true about the sampled urine?

d. It is from a suprapubic tap

443. An 11-year-old is the tallest child in his sixth grade class. He is thin, has a high arched palate, mild scoliosis, and joint hyperextensibility. He is an honor student, and enjoys basketball but must use prescription sports goggles so he can see the ball on the court. He is likely to have which of the following conditions?

d. Marfan syndrome

226. One of your asthmatic patients arrives for a checkup. The mother reports that the child seems to need albuterol daily, especially when exercising, and she has coughing fits that awaken her from sleep about twice a week. Her grandmother had recommended a Chihuahua as a "cure" for her asthma, but her mother has seen no difference since the arrival of the pet. Appropriate treatment measures would include which of the following?

d. Medium-dose, inhaled corticosteroids with short-acting β-agonists

192. During a physical examination for participation in a sport, a 16-yearold girl is noted to have a late apical systolic murmur, which is preceded by a click. The rest of the cardiac examination is normal. She states that her mother also has some type of heart "murmur" but knows nothing else about it. Which of the following is the most likely diagnosis?

d. Mitral valve prolapse

26. A 5-year-old boy presents with the severe rash shown in the photographs. The rash is pruritic, and it is especially intense in the flexural areas. The mother reports that the symptoms began in infancy (when it also involved the face) and that her 6-month-old child has similar symptoms. Which of the following is the most appropriate treatment of this condition?

d. Moisturizers and topical steroids

461. A 4-year-old boy with hyperaldosteronism

d. Na+ 144, K+ 2.9

73. You are called to the delivery room. A newborn infant seems lethargic and has poor tone with only marginal respiratory effort, but his heart rate is above 100 beats per minute. The mother had an uncomplicated pregnancy, and delivery was uncomplicated and vaginal 10 minutes after spontaneous rupture of membranes. The mother received only pain medications while in labor.

d. Naloxone (Narcan)

109. A newborn is noted to be quite jaundiced at 3 days of life. Laboratory data demonstrate his total bilirubin to be 17.8 mg/dL (direct bilirubin is 0.3 mg/dL). Which of the following factors is associated with an increased risk of neurologic damage in a jaundiced newborn?

d. Neonatal sepsis

449. A 4-year-old child with blue sclera and a history of multiple fractures with minimal trauma

d. Normal PO4, normal Ca

451. An otherwise healthy 18-year-old girl with medullary thyroid carcinoma

d. Normal PO4, normal Ca

258. An 8-year-old boy is brought to your office with the complaint of abdominal pain. The pain is worse during the week and seems to be less prominent during the weekends and during the summer. The patient's growth and development are normal. The physical examination is unremarkable. Laboratory screening, including stool for occult blood, CBC, urinalysis, and chemistry panel, yields normal results. Which of the following is the best next step in the care of this patient?

d. Observe the patient and reassure the patient and family

205. A 2-year-old girl is playing in the garage with her Chihuahua, only partially supervised by her father, who is weed-whacking around the garden gnomes in the front yard. He finds her in the garage, gagging and vomiting. She smells of gasoline. In a few minutes she stops vomiting, but later that day she develops cough, tachypnea, and subcostal retractions. She is brought to your emergency center. Which of the following is the most appropriate first step in management?

d. Perform pulse oximetry and arterial blood gas

348. A 2-year-old child is admitted to your hospital team. The child's primary care doctor has been following the child for several days and has noted her to have had high fever, peeling skin, abdominal pain, and a bright red throat. You are concerned because two common pediatric problems that could explain this child's condition have overlapping presenting signs and symptoms. Which of the following statements comparing these two diseases in your differential is true?

d. Pharyngeal culture aids in the diagnosis of one of the conditions.

263. A 16-year-old male, despondent over a recent breakup, tries to commit suicide by taking an unknown quantity of an unknown material he found at home. He is brought to the emergency center by his parents within 30 minutes of the ingestion. For which of the following household materials and medications should he be given activated charcoal as part of his emergency center treatment?

d. Phenobarbital

423. During a routine well-child examination a 10-year-old girl reports that she has occasional headache, "racing heart," abdominal pain, and dizziness. Her mother states that she has witnessed one of the episodes, which occurred during an outing at the mall, and reported the child to be pale and to have sweating as well. Other than some hypertension, she has a normal physical examination. Evaluation of this child is most likely to result in which of the following diagnoses?

d. Pheochromocytoma

120. An infant weighing 1400 g (3 lb) is born at 32 weeks' gestation. Initial evaluation was benign, and the infant was transferred to the level 2 nursery for prematurity. The nurse there calls at 1 hour of life and reports the infant is tachypneic. Vital signs include a heart rate of 140 beats per minute, a respiratory rate of 80 breaths per minute, a temperature of 35°C (95°F), and a peripheral oxygen saturation of 98%. The lungs are clear with bilateral breath sounds and there is no murmur; the infant is in no distress. The child's chest radiograph is shown. Which of the following is the most appropriate next step in evaluating the infant?

d. Place the infant under a warmer.

212. A 6-year-old, fully immunized boy is brought to the emergency room with a 3-hour history of fever to 39.5°C (103.1°F) and sore throat. The child appears alert, but anxious and toxic. He has mild inspiratory stridor and is drooling. He is sitting on the examination table leaning forward with his neck extended. A lateral radiograph of his neck is shown below. Which of the following is the most appropriate immediate management of this patient?

d. Prepare to establish an airway in the operating room.

293. A 6-year-old girl is brought to the emergency room because her urine is red. She has been healthy her whole life, and has recently returned from an outing with her grandmother to a local amusement park. Her urine dip for heme is positive, suggesting which of the following is a possibility?

d. Presence of myoglobin

429. The state laboratory calls your office telling you that a newborn infant, now 8 days old, has an elevated thyroid stimulating hormone (TSH) and low thyroxin (T4) on his newborn screen. If this condition is left untreated, the infant is likely to demonstrate which of the following in the first few months of life?

d. Prolonged jaundice

193. A previously normal newborn infant in a community hospital nursery is noted to be cyanotic at 14 hours of life. She is placed on a face mask with oxygen flowing at 10 L/min. She remains cyanotic, and her pulse oximetry reading does not change. An arterial blood gas shows her PaO2 to be 23 mm Hg. Bilateral breath sounds are present, and she has no murmur. She is breathing deeply and quickly, but she is not retracting. While you are waiting for the transport team from the nearby children's hospital, you should initiate which of the following?

d. Prostaglandin E1 infusion

195. A 15-year-old boy comes to your office for a Special Olympics sports physical. His height is in the 3rd to 5th percentile and his weight is in the 50th percentile. Physical examination reveals a young man with shieldlike chest, cryptorchidism, low-set and malformed ears, ptosis, and pectus excavatum.

d. Pulmonic stenosis

45. A nurse calls you to evaluate an African American newborn whom she thinks has a bacterial skin infection. The areas in question have many scattered pustules full of a milky fluid. Upon examining pustules, they easily wipe away, revealing a small hyperpigmented macule.

d. Pustular melanosis

250. A 2-year-old presents to the emergency center with several days of rectal bleeding. The mother first noticed reddish-colored stools 2 days prior to arrival and has since changed several diapers with just blood. The child is afebrile, alert, and playful, and is eating well without emesis. He is slightly tachycardic, and his abdominal examination is normal. Which of the following is the best diagnostic study to order to confirm the diagnosis?

d. Radionucleotide scan

221. You are asked by a colleague to evaluate a 5-year-old boy as a second opinion. He has a history of chronic and recurrent upper respiratory tract infections, several admissions to the hospital for pneumonia, and three surgeries for PE tubes for chronic otitis media. Of note is a right-sided heart on repeated radiographs. Convinced you know the diagnosis based on history alone, you confirm your diagnosis with a biopsy of the nasal mucosa. You expect to find which of the following?

d. Random orientation of cilia

439. A 14-year-old boy presents with the complaint of "breast swelling." The boy reports that he has been in good health and without other problems, but has noticed over the past month or so that his left breast has been "achy" and that he has now noticed some mild swelling under the nipple. He has never seen discharge; the other breast has not been swelling; and he denies trauma. Your examination demonstrates a quarter-sized area of breast tissue under the left nipple that is not tender and has no discharge. The right breast has no such tissue. He has a normal genitourinary examination, and is Tanner stage 3. Which of the following is the best next course of action?

d. Reassurance of the normalcy of the condition

91. Two days after a backyard party where the children enjoyed limeade and the adults partook of margaritas, a father brings his 4-year-old child to your office for a well-child checkup. The child is healthy other than a slight sunburn and some hyperpigmentation around her face and on her chest. Her father mentions that he, too, has some splotchy hyperpigmentation on his chest

d. Reassurance only

143. You see the newborn baby shown below for the first time in the nursery. You consult plastic and reconstructive surgeon as well as the hospital's speech therapist. Understandably, the parents have many questions. Which of the following statements is appropriate anticipatory guidance for this family?

d. Recurrent otitis media and hearing loss are likely complications.

69. A 2-year-old refuses to walk, has fever, has significant pain with external rotation of the right leg, and has an elevated WBC count.

d. Septic arthritis of the hip

85. A woman without prenatal care has a diet low in green vegetables and enriched grain products.

d. Spina bifida

145. A routine prenatal ultrasound reveals a male fetus with meningomyelocele. The 24-year-old primigravid mother is told the infant will require surgery shortly after birth. You counsel her about the etiology of this defect and the risk of further pregnancies being similarly affected, and state which of the following?

d. Supplementation of maternal diet with folate leads to a decrease in incidence of this condition.

6. A mother brings to your office an article from the Internet suggesting that infants in day care have a statistically higher incidence of upper respiratory infections (p < 0.05) as compared to children not in day care. You explain to her that this means which of the following?

d. The odds are less than 1 in 20 that the differences in upper respiratory infection rates observed were only a chance variation.

206. A 3-year-old girl is admitted with the x-ray shown below. The child lives with her parents and a 6-week-old brother. Her grandfather stayed with the family for 2 months before his return to the West Indies 1 month ago. The grandfather had a 3-month history of weight loss, fever, and hemoptysis. Appropriate management of this problem includes which of the following?

d. Treating the 3-year-old patient with isoniazid (INH) and rifampin

493. A 19-year-old male has recurrent episodes of painful, erythematous, small vesicles, and ulcers on his glans penis.

d. Tzanck preparation for multinucleated giant cells

489. A 19-year-old male college student returns from spring break in Fort Lauderdale, Florida, with complaints of acute pain and swelling of the scrotum. Physical examination reveals an exquisitely tender, swollen right testis that is rather hard to examine. The cremasteric reflex is absent, but there is no swelling in the inguinal area. The rest of his genitourinary examination appears to be normal. A urine dip is negative for red and white blood cells. Which of the following is the appropriate next step in management?

d. Ultrasound of the scrotum

66. A 3-day-old infant who was found at birth to have anal atresia also has vertebral defects, a VSD, tracheoesophageal fistula, absent left kidney, and shortened arms.

d. VATER

Questions 93 to 95 93. A 6-month-old child has a loud systolic murmur at the left lower sternal border

d. Ventricular septal defect (VSD)

100. Breast-fed infant in Alaska

d. Vitamin D

274. An exclusively breast-fed 2-year-old is brought to the emergency center with pain in his right leg after a fall. Physical examination reveals a small child with a 3-cm anterior fontanelle, a flattened occiput, a prominent forehead, significant dental caries, bumpy ribs, and bowed extremities. Radiographs reveal a greenstick fracture at the site of pain, along with fraying at the distal ends of the femur.

d. Vitamin D

24. A child can walk well holding on to furniture but is slightly wobbly when walking alone. She uses a neat pincer grasp to pick up a pellet, and she can release a cube into a cup after it has been demonstrated to her. She tries to build a tower of two cubes with variable success. She is most likely at which of the following age?

e. 1 year

82. Vitamin D

e. A 2-year-old presents with nausea, vomiting, poor feeding, abdominal pain, and constipation. On electrocardiogram (ECG) he has decreased Q-T interval. He has calcifications in his kidneys noted on a CT scan done for his abdominal pain

12. A 4-month-old baby boy arrives to the ER cold and stiff. The parents report that he had been healthy and that they put him to bed as usual for the night at the regular time. When they next saw him, in the morning, he was dead. Physical examination is uninformative. A film from a routine skeletal survey is shown below. Which of the following is the most likely diagnosis?

e. Abuse

314. A previously healthy 7-year-old child suddenly complains of a headache and falls to the floor. When examined in the emergency room (ER), he is lethargic and has a left central facial weakness and left hemiparesis with conjugate ocular deviation to the right. Which of the following is the most likely diagnosis?

e. Acute infantile hemiplegia

220. You receive a telephone call from the mother of a 4-year-old child with sickle-cell anemia. She tells you that the child is breathing fast, coughing, and has a temperature of 40°C (104°F). Which of the following is the most conservative, prudent course of action?

e. Admit the child to the hospital

111. A mother calls you frantic because she has just been diagnosed with chicken pox. She delivered 7 days ago a term infant that appears to be eating, stooling, and urinating well. The child has been afebrile and seems to be doing well. Which of the following is the most appropriate step in management?

e. Advise the mother to continue regular well-baby care for the infant.

498. Delayed menarche and eating disorder

e. An 11-year-old ballerina

209. A 7-year-old child is brought by his mother for a school physical. His growth parameters show his height to be 50th percentile and his weight to be significantly higher than 95th percentile. His mother complains that he always seems sleepy during the day and that he has started complaining of headaches. His second-grade teacher has commented that he has difficulty staying awake in class. His mother complains that he wakes up the whole house with his snoring at night. Which of the following is the most appropriate next step in evaluating and managing this condition?

e. Arrange for polysomnography

355. A 14-year-old boy is seen in the ER because of a 3-week history of fever between 38.3°C and 38.9°C (101°F and 102°F), lethargy, and a 2.7-kg (6-lb) weight loss. Physical examination reveals marked cervical and inguinal adenopathy, enlarged tonsils with exudate, small hemorrhages on the soft palate, a WBC differential that has 50% lymphocytes (10% atypical), and a palpable spleen 2 cm below the left costal margin. Which of the following therapies should be initiated?

e. Avoidance of contact sports

1. Two weeks after a viral syndrome, a 9-year-old boy presents to your clinic with a complaint of several days of weakness of his mouth. In addition to the drooping of the left side of his mouth, you note that he is unable to completely shut his left eye. His smile is asymmetric, but his examination is otherwise normal. Which of the following is the most likely diagnosis?

e. Bell palsy

322. A 6-year-old boy is seen in the office for evaluation of polyuria. Further questioning reveals several months of headache with occasional emesis. Your physical examination reveals a child who is less than 5% for weight. He has mild papilledema. His glucose is normal, and his first urine void specific gravity after a night without liquids is 1.005 g/mL. Which of the following might also be expected to be seen in this patient?

e. Bitemporal hemianopsia

14. A 2-year-old child presents to the office with a paternal complaint of "bowlegs." The girl has always had bowlegs; her previous pediatrician told the family she would grow out of it. Now, however, it seems to be worsening. Her weight is greater than 95% for age, and she has significant bowing out of her legs and internal tibial torsion; otherwise, her examination is normal. A radiograph of her lower leg is shown. Which of the following is the most likely diagnosis?

e. Blount disease

401. A father brings his 3-year-old daughter to the emergency center after noting her to be pale and tired and with a subjective fever for several days. Her past history is significant for an upper respiratory infection 4 weeks prior, but she had been otherwise healthy. The father denies emesis or diarrhea, but does report his daughter has had leg pain over the previous week, waking her from sleep. He also reports that she has been bleeding from her gums after brushing her teeth. Examination reveals a listless pale child. She has diffuse lymphadenopathy with splenomegaly but no hepatomegaly. She has a few petechiae scattered across her face and abdomen and is mildly tender over her shins, but does not have associated erythema or joint swelling. A CBC reveals a leukocyte count of 8,000/μL with a hemoglobin of 4 g/dL and a platelet count of 7,000/μL. The automated differential reports an elevated number of atypical lymphocytes. Which of the following diagnostic studies is the most appropriate next step in the management of this child?

e. Bone marrow biopsy

325. A young infant is noted to have developed constipation over the past week, and then facial diplegia and difficulty sucking and swallowing. The child has been colicky, and the maternal grandmother has been treating the child with a mixture of weak tea, rice water, and honey. Which of the following disorders is the most likely culprit in this child?

e. Botulism

15. A very concerned mother brings a 2-year-old child to your office because of two episodes of a brief, shrill cry followed by a prolonged expiration and apnea. You have been following this child in your practice since birth and know the child to be a product of a normal pregnancy and delivery, to be growing and developing normally, and to have no chronic medical problems. The first episode occurred immediately after the mother refused to give the child some juice; the child became cyanotic, unconscious, and had generalized clonic jerks. A few moments later the child awakened and had no residual effects. The most recent episode (identical in nature) occurred at the grocery store when the child's father refused to purchase a toy for her. Your physical examination reveals a delightful child without unexpected physical examination findings. Which of the following is the most likely diagnosis?

e. Breath-holding spell

207. You are asked to evaluate a 4-year-old boy admitted to your local children's hospital with a diagnosis of pneumonia. The parents state that the child has had multiple, intermittent episodes of fever and respiratory difficulty over the past 2 years, including cyanosis, wheezing, and dyspnea; each episode lasts for about 3 days. During each event he has a small amount of hemoptysis, is diagnosed with left lower lobe pneumonia, and improves upon treatment. Repeat radiographs done several days after each event are reportedly normal. His examination on the current admission is significant for findings similar to those described above, as well as digital clubbing. Which of the following is the most appropriate primary recommendation?

e. Bronchoalveolar lavage

107. A 2-year-old girl has had two episodes of Neisseria meningitidis septicemia and is now admitted for Streptococcus pneumoniae septicemia

e. CBC demonstrating Howell-Jolly bodies

494. A 15-year-old male had a painless papule on his genitals that resolved, but he has now developed a unilateral draining inguinal lymphadenitis

e. Chlamydia culture

157. A week-old infant presents with fever and focal seizure.

e. Congenital herpes simplex virus

370. Two weeks ago, a 5-year-old boy developed diarrhea, which has persisted to the present time despite dietary management. His stools have been watery, pale, and frothy. He has been afebrile. Microscopic examination of his stools is likely to show which of the following?

e. Cryptosporidium

346. A 10-month-old infant on long-term aspirin therapy for Kawasaki disease develops sudden onset of high fever, chills, diarrhea, and irritability. A rapid swab in your office identifies influenza A, adding her to the long list of influenza patients you have seen this December. Over the next few days, she slowly improves and becomes afebrile. However, 5 days after your last encounter you hear from the hospital that she has presented to the emergency center obtunded and posturing with evidence of liver dysfunction. Which of the following statements about her current condition is correct?

e. Death is usually associated with increased intracranial pressures and herniation.

419. An otherwise healthy 7-year-old girl is brought to your office by her father because she has some acne, breast development, and fine pubic hair. Which of the following is the most likely etiology for her condition?

e. Early onset of "normal" puberty (constitutional)

88. A 23-year-old pregnant woman with manic-depressive disorder has had poor prenatal care and was maintained on lithium.

e. Ebstein anomaly

Questions 43 to 48 43. A 1-week-old child's mother complains that the child has a transient rash that has splotchy areas of erythema with a central clear pustule. Your microscopic examination of the liquid in the pustule reveals eosinophils.

e. Erythema toxicum

384. A 3-week-old uncircumcised boy with fever and urinary tract infection

e. Escherichia coli

442. A 3-month-old infant without significant past history was brought to the emergency center by her mother with a generalized tonic-clonic seizure. She is found to have glucose of 5 mg/dL. After correction of her hypoglycemia, she is admitted to your service for further evaluation. Several hours later, her nurse calls to tell you that her bedside glucose check was now 10 mg/dL. You order laboratory work suggested by the pediatric endocrinology team and again correct the infant's hypoglycemia. The results of the laboratory tests you drew include an elevated serum insulin level of 50 μU/mL, and a low IGFBP-1 (plasma insulin-like growth factor binding protein-1). C-peptide levels are not detectable. Which of the following is the likely cause of this child's recurrent hypoglycemia?

e. Factitious hypoglycemia

467. A 2-year-old boy, less than the 5% for weight and height, is in early childhood intervention for developmental delay. He has a short nose, thin upper lip with thin vermillion border, a VSD, and short palpebral fissures

e. Fetal alcohol syndrome

219. A previously healthy 18-month-old has been in a separate room from his family. The family notices the sudden onset of coughing, which resolves in a few minutes. Subsequently, the patient appears to be normal except for increased amounts of drooling and refusal to take foods orally. Which of the following is the most likely explanation for this toddler's condition?

e. Foreign body in the esophagus

398. A 4-year-old previously well African American boy is brought to the office by his aunt. She reports that he developed pallor, dark urine, and jaundice over the past few days. He stays with her, has not traveled, and has not been exposed to a jaundiced person, but he is taking trimethoprim sulfamethoxazole for otitis media. The CBC in the office shows a low hemoglobin and hematocrit, while his "stat" serum electrolytes, blood urea nitrogen (BUN), and chemistries are remarkable only for an elevation of his bilirubin levels. His aunt seems to recall his 8-year-old brother having had an "allergic reaction" to aspirin, which also caused a short-lived period of anemia and jaundice. Which of the following is the most likely cause of this patient's symptoms?

e. Glucose-6-phosphate dehydrogenase deficiency

112. A mother wishes to breast-feed her newborn infant, but is worried about medical conditions that would prohibit her from doing so. You counsel her that of her listed conditions, which of the following is a contraindication to breast-feeding?

e. HIV infection

292. A 4-year-old boy and his family have recently visited a local amusement park. Several of the family members developed "gastroenteritis" with fever and diarrhea, but the 4-year-old's stool was slightly different, as it contained blood. His mother reports that in the past 24 hours he developed pallor and lethargy; she relates that his face looks swollen and that he has been urinating very little. Laboratory evaluation reveals a hematocrit of 28% and a platelet count of 72,000/μL. He has blood and protein in the urine. Which of the following diagnoses is most likely to explain these symptoms?

e. Hemolytic-uremic syndrome

448. A short, 4-year-old, mentally retarded child with brachydactyly of the fourth and fifth digits; obesity with round facies; short neck; subcapsular cataracts; and cutaneous, subcutaneous, and perivascular calcifications of the basal ganglia

e. High PO4, low Ca

450. A 6-year-old child who complains of numbness and tingling of the hands, and later develops tonic-clonic seizures

e. High PO4, low Ca

430. A 1-year-old boy presents with the complaint from his parents of "not developing normally." He was the product of an uneventful term pregnancy and delivery, and reportedly was normal at birth. His previous health-care provider noted his developmental delay, and also noted that the child seemed to have an enlarged spleen and liver. On your examination, you confirm the developmental delay and the hepatosplenomegaly, and also notice that the child has short stature, macrocephaly, hirsutism, a coarse facies, and decreased joint mobility. Which of the following is the most likely etiology of his condition?

e. Hurler syndrome

482. A 15-year-old athlete is in your office for his annual physical examination before the start of football season. He has no complaints, has suffered no injuries, and appears to be physically fit. On his heart examination, you note a heart rate of 100 beats per minute, and a diffuse point of maximal impulse (PMI) with a prominent ventricular lift. He has a normal S1 and S2, with an S4 gallop. He has no murmur sitting, but when he stands you clearly hear a systolic ejection murmur along the lower left sternal edge and the apex. For which of the following conditions is this examination most consistent?

e. Hypertrophic cardiomyopathy

57. A 14-year-old on a mountain-climbing expedition in December becomes tired, clumsy, and begins to hallucinate. His heart rate is 45 beats per minute

e. Hypothermia

373. A patient presents to the emergency center with a 6-hour history of fever to 38.9°C (102°F). Her mother reports that the patient appeared to be feeling poorly, that she had been eating less than normal, and that she vomited once. About 2 hours prior to arrival at the ER, the mother states that she noted a few purple spots scattered about the body on the patient, especially on the buttocks and legs. On the 30-minute ride to the ER, the purple areas spread rapidly and became coalesced in areas, and the patient is now obtunded. Which of the following is the most likely diagnosis?

e. Meningococcemia

Questions 275 to 277 275. A 3-week-old child currently admitted to the hospital for pneumonia who gags and chokes during feedings

e. Modified barium swallow

217. A 13-year-old develops fever, malaise, sore throat, and a dry, hacking cough over several days. He does not appear to be particularly sick, but his chest examination is significant for diffuse rales and rhonchi. The chest radiograph is shown below. Which of the following is the most likely pathogen?

e. Mycoplasma pneumoniae

459. An 11-year-old boy with central diabetes insipidus secondary to an automobile accident

e. Na+ 155, K+ 5.5

460. A 2-year-old girl with nephrogenic diabetes insipidus

e. Na+ 155, K+ 5.5

185. A newborn infant has mild cyanosis, diaphoresis, poor peripheral pulses, hepatomegaly, and cardiomegaly. Respiratory rate is 60 breaths per minute, and heart rate is 250 beats per minute. The child most likely has congestive heart failure caused by which of the following?

e. Paroxysmal atrial tachycardia

353. A 15-month-old boy is brought to the ER because of fever and a rash. Six hours earlier he was fine, except for tugging on his ears; another physician diagnosed otitis media and prescribed amoxicillin. During the interim period, the child has developed an erythematous rash on his face, trunk, and extremities. Some of the lesions, which are of variable size, do not blanch on pressure. The child is now very irritable, and he does not interact well with the examiner. Temperature is 39.5°C (103.1°F). He continues to have injected, immobile tympanic membranes, but you are concerned about his change in mental status. Which of the following is the most appropriate next step in the management of this infant?

e. Perform a lumbar puncture

162. An infant of uncertain but seemingly term dates is born via emergent cesarean section for nonreassuring heart tones; the obstetrician has noted little or no amniotic fluid. The infant is small, has abnormally shaped limbs, and an unusual facies. The child has immediate respiratory distress. A chest radiograph reveals a poorly developed chest with little lung tissue.

e. Primary pulmonary hypoplasia

27. A 1-year-old presents for a well-child checkup, but the parents are concerned about giving the child his immunizations. Which of the following is a true contraindication to the administration of the fourth DTaP (diphtheria and tetanus toxoid and acellular pertussis) vaccine?

e. Prolonged seizures 6 days after the last DTaP vaccine

455. A 5-day-old girl with brachydactyly, round facies, and short neck

e. Pseudohypoparathyroidism

420. The parents of a 14-year-old boy are concerned about his short stature and lack of sexual development. By history, you learn that his birth weight and length were 3 kg and 50 cm, respectively, and that he had a normal growth pattern, although he was always shorter than children his age. The physical examination is normal and his growth curve is shown on the next page. His upper-to-lower segment ratio is 0.98. A small amount of fine axillary and pubic hair is present. There is no scrotal pigmentation; his testes measure 4.0 cm3 and his penis is 6 cm in length. In this situation, which of the following is the most appropriate course of action?

e. Reassure the parents that the boy is normal

22. You are performing a well-child examination on the 1-year-old child shown in the picture. For this particular problem, which of the following is the most appropriate next step in management?

e. Refer immediately to ophthalmology

119. The nurse from the level 2 neonatal intensive care nursery calls you to evaluate a baby. The infant, born at 32 weeks' gestation, is now 1 week old and had been doing well on increasing nasogastric feedings. This afternoon, however, the nurse noted that the infant has vomited the last two feedings and seems less active. Your examination reveals a tense and distended abdomen with decreased bowel sounds. As you are evaluating the child, he has a grossly bloody stool. The plain film of his abdomen is shown. The next step in your management of this infant should include which of the following?

e. Removal of nasogastric tube, placement of a transpyloric tube and, after confirmation via radiograph of tube positioning, switching feeds from nasogastric to nasoduodenal.

313. A 14-year-old girl with a history of seizures is admitted to the hospital with the diagnosis of status epilepticus. Her valproic acid level is in the therapeutic range. You arrange a 24-hour video electroencephalogram (EEG). During the EEG, she has several episodes of tonic and clonic movements with moaning and crying, with no loss of bowel or bladder control. The neurologist tells you that during the events the EEG had excessive muscle artifact but no epileptiform discharges. Which of the following treatments is the most appropriate for this condition?

e. Request a psychiatric evaluation

216. An 8-year-old girl presents with well-controlled, moderately persistent asthma. Her therapies consist of occasional use of short-acting β-agonists, daily inhaled steroids, and a leukotriene inhibitor. She presents with white patches on her buccal mucosa. You recommend which of the following?

e. Rinse her mouth after use of her inhaled medications

367. An 8-year-old Cub Scout who returned from an outing 9 days ago is brought to the clinic with the rapid onset of fever, headache, muscle pain, and rash. The maculopapular rash began on the flexor surfaces of the wrist and has become petechial as it spread inward to his trunk. Which of the following is the most likely diagnosis?

e. Rocky Mountain spotted fever

347. An 18-month-old child presents to the emergency center having had a brief, generalized tonic-clonic seizure. He is now postictal and has a temperature of 40°C (104°F). During the lumbar puncture (which ultimately proves to be normal), he has a large, watery stool that has both blood and mucus in it. Which of the following is the most likely diagnosis in this patient?

e. Shigella

74. A 4-year-old girl comes into the ER after eating a bottleful of small, chewable pills she found while at her grandfather's house. She has an increased respiratory rate, elevated temperature, vomiting, and is disoriented. She is intermittently complaining that "a bell is ringing" in her ears. She has a metabolic acidosis on an arterial blood gas.

e. Sodium bicarbonate

51. Parents hear over their baby monitor that their 5-year-old girl regularly calls out during the night. When the parents check on her, she is sleeping comfortably and is in no apparent distress.

e. Somniloquy

248. A 4-year-old boy, recently adopted through an international adoption service, is noted to have intermittent watery diarrhea, nausea, belching, and abdominal pain. His weight is less than the fifth percentile for his age. Which of the following studies would be most helpful in making the diagnosis?

e. Stool microscopy for ova and parasites

Questions 172 to 174 172. A 1-day-old infant has a fronto-occipital head circumference that is 2 cm larger than the initial measurement done several hours before, the scalp has a "squishy" feel to it, and the infant has developed tachycardia.

e. Subgaleal hemorrhage

202. A 10-month-old infant has poor weight gain, a persistent cough, and a history of several bouts of pneumonitis. The mother describes the child as having very large, foul-smelling stools for months. Which of the following diagnostic maneuvers is likely to result in the correct diagnosis of this child?

e. Sweat chloride test

35. A 14-year-old high school student arrives to your clinic for well-child care. In reviewing his records you determine that his most recent immunization for tetanus was at 4 years of age. Which of the following should you recommend?

e. Tetanus toxoid, reduced diphtheria toxoid, and acellular pertussis vaccine adsorbed (Tdap)

311. On a newborn boy's first examination, you note a prominent occiput, a broad forehead, and an absent anterior fontanelle. The baby's head is long and narrow. The remainder of the physical examination, including a careful neurological evaluation, is normal. You note that the baby was born via cesarean section for cephalopelvic disproportion. When you enter the mother's room, the first question she asks is about her baby's head shape. Which of the following is the most appropriate statement to the mother about this infant's condition?

e. The condition requires referral to a surgeon.

395. The parents of a previously healthy 2-year-old child note her to be pale and bring her to your clinic for evaluation. She currently has no fever, nausea, emesis, bone pain, or other complaints. Her examination is significant for pallor, tachycardia, and a systolic ejection murmur, but she has no organomegaly. Her complete blood count (CBC) reveals a hemoglobin of 4 g/dL, normal indices for age, a WBC count of 6.5/μL, and a platelet count of 750,000/μL. Her reticulocyte count is 0%. Coombs test is negative. Her peripheral blood smear shows no blast forms and no fragments. Red blood cell (RBC) adenosine deaminase levels are normal. A bone marrow reveals markedly decreased erythroid precursors. Which of the following is this child's likely diagnosis?

e. Transient erythroblastopenia of childhood

70. A 3-year-old refuses to walk, is afebrile, had an upper respiratory tract infection a week ago, has right hip pain with movement, and has a normal WBC count.

e. Transient synovitis

146. A term, 4200-g (9-lb, 4-oz) female infant is delivered via cesarean section because of cephalopelvic disproportion. The amniotic fluid was clear, and the infant cried almost immediately after birth. Within the first 15 minutes of life, however, the infant's respiratory rate increased to 80 breaths per minute, and she began to have intermittent grunting respirations. The infant was transferred to the level 2 nursery and was noted to have an oxygen saturation of 94%. The chest radiograph is shown. Which of the following is the most likely diagnosis?

e. Transient tachypnea of the newborn

372. An 8-year-old immigrant from rural Central America presents with complaints of weakness, facial swelling, muscle pain, and fever. A CBC reveals marked eosinophilia. Which of the following parasites is most likely to be responsible?

e. Trichinella spiralis

473. A previously normal father of one of your patients, from a family with members known to have fragile X syndrome, develops ataxia and tremor

e. Triplet repeat expansion disorder

102. 1-day-old newborn

e. Vitamin K

441. An 18-year-old girl has hepatosplenomegaly, an intention tremor, dysarthria, dystonia, and deterioration in her school performance. She also developed abnormal urine with excess glucose, protein, and uric acid. She has a several-year history of elevated liver enzymes of unknown etiology. Which of the following best explains her condition?

e. Wilson disease

500. Shoulder subluxation, knee injuries, and dermatologic problems such as herpes simplex, impetigo, and staphylococcal furunculosis or folliculitis

f. A 16-year-old high school wrestling champion

101. Sickle-cell disease

f. Folate

97. Administration of phenytoin

f. Folate

385. Several weeks' history of epigastric and periumbilical pain that worsens with fasting and is relieved by eating, in a 9-year-old with guaiac-positive stools

f. Helicobacter pylori

Questions 303 to 306 303. An 8-year-old boy with the intermittent complaint of "burning" when he urinates and who has trace blood on his urine dip test.

f. Idiopathic hypercalciuria

270. A 9-month-old infant, who has been fed cow's milk exclusively for 4 months, is tachycardic and pale.

f. Iron

267. A 7-year-old has been vomiting for 2 days and has had diarrhea for 1 day. He now notes that he has small streaks of blood in his emesis. The rest of his family has had similar symptoms

f. No immediate intervention

104. A 5-month-old infant is admitted with severe varicella infection. The lesions cover the infant's entire body, and the infant is beginning to show symptoms of respiratory distress. Past medical history is significant for a history of atopic dermatitis. The family also notes frequent epistaxis; the last episode required nasal packing in the ED.

f. Platelet count

161. A postterm infant is born at home after a prolonged and difficult labor. The maternal grandmother brings the infant to the hospital at 1 hour of life because of fast breathing. Grandmother notes that the child seemed well for a while, but then developed increased work of breathing. Physical examination reveals an infant in moderate respiratory distress with diminished breath sounds on the left. Chest radiograph reveals the heart to be pushed to the right side and loss of lung markings in the left lung field.

f. Pneumothorax

456. A 15-year-old girl with menstrual irregularities and hirsutism

f. Polycystic ovary syndrome

86. A woman with long-standing hypertension treated with angiotensinconverting enzyme (ACE) inhibitors

f. Renal dysgenesis

48. A newborn's father complains that his son has dandruff, with many waxy flakes of skin on the scalp. When he scrapes the lesions, hair often comes off with the flakes of skin. In addition, the baby has flaking of the eyebrows.

f. Seborrheic dermatitis

90. A 2-week-old boy is brought by his mother to the clinic; he has scaly, yellow patches on his scalp with associated hair loss.

f. Topical steroids or a selenium sulfide-containing product

468. A 14-year-old girl with short stature, thick neck, minimal pubertal development, repaired coarctation of the aorta, and normal intelligence.

f. Turner syndrome

Questions 59 to 62 59. Megaloblastic anemia, growth failure, paresthesias, sensory defects, developmental regression, weakness, and fatigue

f. Vitamin B12 deficiency

499. Hyperextension of the thumb and sprains of the anterior cruciate ligament

g. A 14-year-old snow skier

80. Nicotinic acid

g. After getting into his mother's bottle of vitamins, a 3-year-old has burning, tingling, and itching on his arms as well as a reddened face, arms, and chest.

87. A primiparous mother late in her first trimester has a fever and "3-day" measles.

g. Cataracts

Questions 464 to 470 464. A 10-year-old boy with hypermobile joints and poor wound healing

g. Ehlers-Danlos syndrome

106. A general practitioner refers to you for evaluation a 3-year-old boy with frequent infections. You note the child to have a loud systolic murmur, posteriorly rotated ears that are small and low-set, down-slanting and widely spaced eyes, a small jaw, and an upturned nose. At birth the child spent 2 weeks in the nursery for "low calcium" and seizures, and he still receives calcium supplementation, but the mother does not know why. You would like to order a rapid diagnostic test for this child.

g. Intradermal skin test using Candida albicans

47. A newborn's mother complains that her infant seems to have very small white dots all over his nose. The dots do not wipe off with bathing, but they are also not erythematous

g. Milia

391. A 2-month-old boy with a 3-day history of upper respiratory infection who suddenly develops high fever, cough, and respiratory distress; within 48 hours, the patient has developed a pneumatocele and a left-sided pneumothorax.

g. Staphylococcal pneumonia

Questions 452 to 457 452. A 15-year-old defensive lineman for his high school football team whose mother reports that his shoulder pads have permanently stained his neck

g. Type 2 diabetes

61. Irritability, convulsions, hypochromic anemia

g. Vitamin B6 deficiency

271. A 3-day-old infant born at home is brought to the emergency center with bloody stools, hematemesis, and purpura. His circumcision is oozing blood.

g. Vitamin K

Questions 380 to 385 380. An 18-year-old college girl with an extremely sore throat and high fever who develops a rash upon administration of ampicillin

i. Epstein-Barr virus

Questions 160 to 163 160. A large-for-gestation-age term infant is delivered via scheduled cesarean section develops, at 15 minutes of age, tachypnea, grunting, flaring, and retractions. The child does not move his left arm well, but you find no clavicular fracture. A chest radiograph shows the left diaphragm to be markedly higher than the right.

i. Phrenic nerve paralysis

60. Photophobia, blurred vision, burning and itching of eyes, poor growth, cheilosis

i. Riboflavin deficiency

177. You are seeing a 2-year-old boy for the first time. His father denies any past medical or surgical history, but does note that the child's day care recently sent a note home asking about several episodes, usually after the child does not get what he wants, when he "breathes funny" and sits in a corner with his knees under his chin for a few minutes. The day-care staffers think this "self-imposed time-out" is a good thing, but they worry about the breathing. One teacher even thought he once looked blue, but decided that it was probably because of the finger paints he had been using. On examination, you identify a right ventricular impulse, a systolic thrill along the left sternal border, and a harsh systolic murmur (loudest at the left sternal border but radiating through the lung fields). His chest radiograph and ECG are shown. Which of the following congenital cardiac lesions would you expect to find in this child?

אין תשובה שלך ואין תשובות של הספר


Ensembles d'études connexes

Chapter 53: Assessment of Kidney and Urinary Function PREP U RWACH MASTERY LEVEL 3 MINIMUM

View Set

GEB module 5 quiz (chapter 5, 13, and 14)

View Set

Chapter 6 - Accounting for Merchandising Businesses

View Set

Business Ethics Concepts and Cases Ch.1

View Set

Physical Examination of the Dog and Cat

View Set

Canada's Provinces/territories and capitals from west to east

View Set